Contabilidad Tercero Básico

May 25, 2019 | Author: Magdiel Ochoa | Category: Accounting, Budget, Taxes, Banks, Money
Share Embed Donate


Short Description

Planificación Contabilidad Tercero Básico....

Description

José Antonio Tzilin Herrera Rudy Edgar Ramos Ortiz Soyla Luz de la Esperanza Arango de Valiente Silvia Verónica López Gramajo Leandro Jeremías González González Heriberto Santiago Guox de la Cruz Rodolfo Avelino Túnchez García

Planificación de los aprendizajes CONTABILIDAD TERCERO BÁSICO

22- Noviembre- 2011 Mazatenango, Suchitepéquez

PLANIFICACIÓN DE LOS APRENDIZAJE Establecimiento educativo: Instituto Nacional de Educación Básica INEB Ciclo Escolar: 2012 Grado: 3ro Básico Sección: Jornada: Vespertina Nombre del docente: ______________________________________ AREA: Productividad y Desarrollo (subárea) Contabilidad Tema/Unidad: Primera Tiempo: 2 períodos

Formula Proyectos viables que propician el mejoramiento escolar o comunitario. CONTENIDOS Declarativos Regímenes de Tributación.

Procedimentales Caracterización del Tributario Nacional.

Actitudinales Sistema

Definición. Impuestos Formularios Comparación entre las ventajas y desventajas de los regímenes Tributarios para los contribuyentes

Sistema Tributario Nacional Concepto de Impuestos

Manifiesta interés en el cumplimiento de las normas tributarias de las profesiones como fuente de desarrollo del entorno.

Clasificación de Impuestos. Tarifas de Impuestos Tipos de Contribuyentes Educación Fiscal

: Establece las profesiones que generan desarrollo social y económico en el contexto.

Procedimiento sugerido para el desarrollo de la competencia No.1 Con las actividades diseñadas para el logro de esta competencia, las y los estudiantes incorporarán a sus estructuras mentales, la capacidad de análisis e interpretación de los diferentes tributos y contribuciones del Sistema Tributario Nacional. Los Tributos deben estar basados en los siguientes principios generales: Efectividad, Eficiencia, Equidad y Sencillez. De conformidad con el artículo 135 de la Constitución Política de la República de Guatemala son deberes de los guatemaltecos entre otros contribuir con los gastos públicos, en forma prescrita por la ley y a obedecer las leyes. De aquí se deriva el poder Tributario, corresponde al Congreso de la República decretar, reformar y derogar las leyes. (1)

Regímenes de Tributación y Sistema Tributario Inicialmente, la o el docente, establece la relación de Investigación bibliográfica y exposición, organizando equipos de trabajo de 5 a 7 alumnos, designando los siguientes temas: 1.- Concepto de los diferentes Impuestos actuales. 2.- Clasificación de Impuestos. 3.- Tarifas de Impuestos Municipal y Estatal. 4.- Tipos de Contribuyentes. 5.- Educación Fiscal.

Durante las exposiciones el docente promueve la participación de las y los estudiantes por medio de cuestionamientos entre los diferentes equipos de trabajo, dando así respuesta a las inquietudes sobre los temas de exposición. representa la mayor parte de recursos del Estado, puesto que todos aportamos para hacer posibles los servicios públicos que directa o indirectamente benefician al conjunto de habitantes. El Tributo constituye la cantidad en dinero o equivalentes, que los habitantes deben pagar al Estado. El Tributo es tan antiguo como la sociedad misma, hoy el Tributo es un ingrediente fundamental para la producción de bienes públicos, que son necesarios para el fortalecimiento de la economía de mercado.

Sistema Tributario Nacional es el conjunto de impuestos que se aplican en el país y que obedece a sus políticas fiscales y tributarios. Superintendencia de Administración Tributaria (SAT): Es una entidad que fue creada en 1998 para recaudar, fiscalizar, administrar el régimen tributario y controlar el sistema aduanero. Tiene competencia y jurisdicción en todo el país; tiene autonomía funcional, financiera y administrativa. El Impuesto es el tributo que recibe el Estado para realizar actividades no relacionadas con el contribuyente. Los Impuestos se clasifican en: Directos e Indirectos (1) Impuestos Directos: son aquellos que recaen sobre las personas y los bienes, entre estos tenemos los siguientes impuestos: 1) Impuesto Sobre la Renta (ISR): grava la riqueza en su manifestación inmediata, recae sobre las rentas o utilidades que provengan de inversiones de capital, de trabajo o ambos. 2) Impuesto de Solidaridad (ISO) este impuesto fue autorizado en sustitución sobre los acuerdos de paz. El cálculos se hace tomando el total de las ventas del año y se divide entre 4 (que son los trimestres del año), y el resultado se multiplica por el 1%, siendo el resultado obtenido el impuesto a pagar. 3) Impuesto Único sobre Inmuebles(IUSI): Es un impuesto anual cuyo hecho generador es la propiedad o posesión de un bien inmueble, comprendiendo terrenos, construcción en el área urbana .

Impuestos Indirectos: recaen directamente sobre el consumo, entre ellos podemos mencionar: 1) Impuesto al Valor Agregado: Es el impuesto que se paga en las operaciones que se realizan en la compra-venta o permuta de bienes muebles o inmuebles, la prestación de servicios, las importaciones, arrendamientos. El cual es pagado por el comprador.

Existen cuatro tipos de Contribuyentes, los cuales son: 1) Pequeño Contribuyente: son las personas o empresas que sus ingresos anuales no superan los Q 60,000 2) Persona Jurídica: son las empresas, organizaciones, instituciones, asociaciones, que tienen derechos y obligaciones ante las leyes. 3) Profesional Liberal: es la personan que posee un título universitario, trabaja por cuenta propia y percibe sus ingresos por medio de honorarios. 4) En Relación de Dependencia: son todos los empleados que prestan sus servicios a una empresa, entidad, organismo o dependencia, a cambio de un sueldo o salario. (2) Concluidas las exposiciones la o el docente explicará mediante una clase magistral los temas asignados a los equipos de trabajo.

RECURSOS DIDÁCTIVOS Para la actividad No. 1

Carteles, Pizarrón, Bibliográficas, Otros

Marcadores,

Sellador,

Hojas,

Fichas

Luego del desarrollo de las actividades sugeridas y aquellas diseñadas por usted, se sugiere que evalúe el proceso de aprendizaje, para verificar el nivel de alcance de la competencia No. 1

Autoevaluación: Las y los estudiantes se autoevalúan, con el propósito de formarse un juicio de valor acerca del nivel de aprendizaje alcanzado. El docente puede utilizar las estrategias que respondan a este propósito. Por ejemplo: Coevaluación. Para que autoevaluación se obtenga alcanzado mejores resultados, necesario las y elaborar los estudiantes evalúen unos con la otros, el nivel de aprendizaje durante elesproceso. Seque sugiere un instrumento para evaluar este proceso: Lista de cotejo, escala de rango, etc. Los indicadores deben ser claros para no confundir al estudiante.

Heteroevaluación. Es el procedimiento que realiza el docente para evaluar el nivel de aprendizaje de las y los estudiantes, considerando la auto y coevaluación. Para evaluar la actividad que realizaron de presentación oral, se sugiere utilizar la siguiente lista de cotejo. Además, puede realizar momentos de evaluación individual y en equipo de trabajo.

Instrucciones: marque con una X los aspectos que demostró cada estudiante durante la presentación oral. Deje en blanco las casillas de aquellos aspectos no observados.

Nombre estudiante

del a l e d n ió c a t n e s re P

n ió c a g it s e v n I

a m te l e ó t n e s re P

n o s s e n io s e r p x e s a L

s te n e in tr e p

o d a z ilit u l ria te a m l E

o d a u c e d a e u f

s a e id s a v e u n ó tr o p A

r a s e r te in a i c ió e n i u d ig u s a n a o l C a

e d n o p s e R

s a l

e fu

e t n e m a d a rt e c a

o id n te n o c

s a t n u g e r p

e t n e i ic f l u E s

o id n te b o e j a t n u P

Fuentes: 1. Contabilidad General de Tercero Básico EDITEXSA, páginas 142, 145 2. Folleto proporcionado por la SAT, EDUSAT, páginas 1-10 3. Ley del Impuesto Al Valor Agregado (IVA) Decreto 27-92 4. Ley del Impuesto Sobre La Renta (ISR) Decreto 26-92 5. Ley del Impuesto Único Sobre Inmuebles (IUSI)

Establecimiento educativo: Instituto Nacional de Educación Básica INEB Ciclo Escolar: 2012 Grado: 3ro Básico Sección: Jornada: Vespertina Nombre del docente: ______________________________________ AREA: Productividad y Desarrollo (subárea) Contabilidad Tema/Unidad: Primera Tiempo: 2 períodos

Aplica técnicas de mejora continua en el desarrollo de procesos productivos en su entorno.

CONTENIDO DECLARATIVO Presupuestos de una empresa: .- Metodología presupuestal.

PROCEDIMENTALES

ACTITUDINALES

Formulación de presupuestos de procesos empresariales.

. Concepto

Manifiesta responsabilidad en el control de la calidad en el desarrollo de procesos productivos.

. Tipos de presupuesto. . Origen . Integración Estimación de costos de un .ejecución. Preparación, elaboración y

proceso productivo.

. Análisis de las variaciones presupuestales. . Técnicas de registro.

Planifica sistemas de control para el desarrollo de procesos productivos. Procedimiento sugerido para el desarrollo de la competencia No. 2 Con las actividades diseñadas para el logro de esta competencia, las y los estudiantes incorporarán a sus estructuras mentales, conocimientos acerca del Presupuesto de una empresa, su metodología, tipos de presupuesto, preparación elaboración y ejecución. El presupuesto se convierte en una herramienta necesaria de dirección dentro de la organización y administración de la familia, negocio, empresa y el gobierno la elaboración, preparación y ejecución se manifiesta de la siguiente manera: familiar, comercial y pública. (6)

Diferentes presupuestos, concepto, principios del presupuesto.

Inicialmente, la o el docente, lleva a cabo una lluvia de ideas, para que las y los estudiantes personalmente, expresen los conocimientos previos que tienen con relación a: ¿Qué es Ingreso? ¿Qué es Egreso o Gasto? ¿A qué se refiere la palabra FIJO? ¿A qué se refiere la palabra VARIABLE? ¿Qué es un Presupuesto? Durante la realización de la lluvia de ideas expresadas por los estudiantes el docente realizara un mapa conceptual sobre los términos anteriores, con la finalidad de socializar el tema. Concluida la lluvia de ideas, la o el docente explica en clase magistral los temas antes mencionados.

El Presupuesto es un plan de acción cuantitativo y un auxiliar de la coordinación y el control. Los presupuestos básicamente, no son sino Estados Financieros anticipados, expresiones formales de los planes administrativos. El presupuesto es un proyecto detallado de los resultados de un programa oficial de operaciones, basado en una eficiencia razonable. EL PRESUPUESTO es el conocimiento anticipado de las operaciones futuras de la empresa, empleando métodos técnicos y científicos que abarquen a la empresa como un todo.

Principios d

el Presupuesto

:

1) Previsión: En función de los objetivos de la empresa deben preverse todas las implicaciones de los planes y cuantificarse. 2) Participación participar, en el proceso de la elaboración del Presupuesto los jefes responsables :deDeben su ejecución. 3) Patrocinio: Debe ser patrocinado por la administración.

4) Responsabilidad: Debe definirse con precisión la responsabilidad de los jefes en la ejecución presupuestal que les corresponda. 5) Realismo: Las cifras del presupuesto deben representar metas razonablemente obtenibles, ni tan altas que no pueden lograrse y srcinen frustraciones, ni tan bajas que propicien las complacencias.

6) Aprobación: La Aprobación final del presupuesto debe ser específica y comunicarse a todo el organismo. 7) Costeabilidad: Las ventajas que se obtengan como consecuencia del sistema de control presupuestal establecido, han de superar el costo mismo de instalación y funcionamiento del sistema. 8) Flexibilidad: Debe mostrarse el comportamiento esperado de los costos a varios niveles de producción o venta. (6)

Los Ingresos son los recursos, de carácter económico y en especie que obtienen determinados miembros del núcleo familiar. Los Ingresos Fijos son las entradas de dinero que tiene una familia y que son constantes en tiempo y cantidad. Los Ingresos Variables son las que tiene una familia y que no son constantes ni en tiempo ni en cantidad. Los Egresos son todos los gastos en que incurren las familias o entidades comerciales en la satisfacción de sus necesidades. Los Egresos Fijos son los gastos de dinero que tiene una familia y que son constantes en tiempo y cantidad. Los Egresos Variables son los gastos de dinero que tiene una familia o entidad comercial y que no son constantes en tiempo y cantidad, y comprenden impuestos sobre agua, energía eléctrica, gas, teléfono, medicina, recreación, etc. (7)

Autoevaluación: Las y los estudiantes se autoevalúan, con el propósito de formarse un juicio de valor acerca del nivel de aprendizaje alcanzado. El docente puede utilizar las estrategias que respondan a este propósito. Por ejemplo: Puede efectuar un debate para que las y los alumnos se intercambien sus conocimientos adquiridos.

Coevaluación. Para autoevaluación se obtenga alcanzado mejores resultados, necesario las y elaborar los estudiantes evalúen unos que con la otros, el nivel de aprendizaje durante elesproceso. Seque sugiere un instrumento para evaluar este proceso: Lista de cotejo, escala de rango, etc. Los indicadores deben ser claros para no confundir al estudiante.

Heteroevaluación. Es el procedimiento que realiza el docente para evaluar el nivel de aprendizaje de las y los estudiantes, considerando la auto y coevaluación. Para evaluar la actividad que realizaron el docente realiza un cuestionario a todos los estudiantes para verificar lo aprendido

Presentación de Modelo de presupuesto por parte del docente. El docente presenta a los estudiantes el siguiente ejercicio para resolverlo en clase para que ellos tengan los lineamientos a seguir para la elaboración de un presupuesto comercial. EMPRESA COMERCIAL LA LIBERTAD, S.A. PRESUPUESTO DE GASTOS MENSUALES. MES DE DICIEMBRE. 1.- Ingreso: Promedio de venta de mercadería durante 6 meses de Q80,000.00 2.- Gastos Fijos: Luz Q1,260.00, AguaQ300.00 , Telefono Q825.00, Internet Q350.00, Alquiler Q6,000.00, Utensilios de limpieza Q250.00, Pago de empleados (conserje Q1,500.00, Guardian Q2,000.00, Secretarias Q5,000.00, Dependientes Q9,000.00, piloto Q2,500.00). 3.- Gastos Variables: GasolinaQ2,550.00 , publicidad Q1,000.00, reparaciones Q3,000.00 4.- Inversiones: En mercadería Q25,000.00, Mobiliario y equipo Q1,200.00) EMPRESA COMERCIAL LA LIBERTAD, S. A. PRESUPUESTO DE GASTOS MENSUALES DICIEMBRE DE 2011. ( Expresado en Quetzales ) GASTOS GASTOS FIJOS Luz

1260

Agua

300

Teléfono

825

Internet

350

Alquiler

6,000

Utensilios de Limpieza

250

EMPLEADOS Conserje

1,500.00

Guardián

2,000.00

Secretarias

5,000.00

Dependientes

9,000.00

Piloto

2,500.00

20,000.00

GASTOS VARIABLES Gasolina

2550

Publicidad

1000

Reparaciones

3000

INVERSIONES Mercadería

25,000.00

Mobiliario y Equipo

1200

Total Egresos INGRESOS

80,000.00

EGRESOS

61,735.00

DISPONIBLE

18,265.00

Guatemala 31 de Diciembre de 2,011.

(8)

61735

Firma:____________________________

Vo.bo.____________

P.C. José Horacio González García.

Propietario.

Autoevaluación: Las y los estudiantes se autoevalúan, con el propósito de formarse un juicio de valor acerca del nivel de aprendizaje alcanzado. El docente puede utilizar las estrategias que respondan a este propósito. Por ejemplo: Que el alumno exprese los conocimientos adquiridos, anotándolos en su cuaderno y posteriormente el docente revisará la anotación realizada por el alumno.

Heteroevaluación. Es el procedimiento que realiza el docente para evaluar el nivel de aprendizaje de las y los estudiantes. Para evaluar los conocimientos prácticos adquiridos, se sugiere utilizar como modelo el ejemplo visto en clase. Para este tipo de evaluación se le recomienda que realice equipos de trabajo de 5 a 7 alumnos y

les asigne un ejercicio diferente de Presupuesto a cada equipo de trabajo.

EJERCICIO PROPUESTO: EMPRESA COMERCIAL MULTIREPUESTOS, S.A. PRESUPUESTO GASTO MENSUAL. MES DE NOVIEMBRE DE _________. Promedio de venta de mercadería durante 6 meses de Q160,000.00 En mercadería Q50,000.00, Mobiliario y equipo Q2,400.00 Luz Q2,400.00, AguaQ600.00 , Teléfono Q825.00, Internet Q350.00, Alquiler Q10,000.00, Utensilios de limpieza Q500.00, Pago de empleados (conserje Q5,000.00, Guardián Q4,000.00, Secretarias Q10,000.00, Dependientes Q9,000.00, piloto Q2,500.00). GasolinaQ2,550.00, publicidad Q1,000.00, reparaciones Q3,000.00

Para la actividad No. 1 Para la actividad No.2

Pizarra, marcadores, yeso, pliego de papel de 80 gramos. Pizarra, marcadores, cuadernillos, fotocopias, carteles, sellador.

Fuentes: 6. Contabilidad General de Tercero Básico, EDITEXSA, pagina 143 7. Contabilidad General de Tercero Básico, Editora Educativa, página 19 8. Libro de Santillana.

Establecimiento educativo: Instituto Nacional de Educación Básica INEB Ciclo Escolar: 2012 Grado: 3ro Básico Sección: Jornada: Vespertina Nombre del docente: ______________________________________ AREA: Productividad y Desarrollo (subárea) Contabilidad Tema/Unidad: Numeral 1) Primera Unidad Tiempo: 9 períodos Numeral 2) Primera Unidad Tiempo: 7 períodos Numeral 3) Segunda Unidad Tiempo: 15 períodos Numeral 4) Tercera Unidad Tiempo: 15 períodos Numeral 5) Cuarta Unidad Tiempo: 10 períodos Numeral 6) Cuarta Unidad Tiempo: 2 períodos

Ejecuta propuestas de emprendimiento productivo escolar o comunitario que enfatizan la integración de liderazgo y procesos administrativo y comerciales. CONTENIDO DECLARATIVO

PROCEDIMENTALES

ACTITUDIN ALES

1.Contabilidad y Técnicas de Registros en libros de Contabilidad 1.1.Concepto. 1.2.Principios de contabilidad. Caracterización de cada uno de los 1.3.Tipos de contabilidad. libros de contabilidad. 1.4.Base Legal. 1.5.Libros Obligatorios. 1.6.Libros Auxiliares de Contabilidad. 1.7. Técnica de Registro. 1.8.Terminología contable. 2. Apertura de la Contabilidad. 2.1.Inventario 2.2. Concepto 2.3. Integración. 2.4. Ejecución 2.5. Balance General de Apertura 3. Libro Diario y Libro Caja. 3.1.Jornalizar 3.2. Partidas Simples 3.3. Partidas Mixtas. 3.4. Partidas Compuestas. 4. Libro mayor o centralizador. 4.1.”T” Grafica. 4.2. Doble folio. 4.3. Un folio. 4.4. Libros Auxiliares del Mayor. 4.4.1. Libro de compras. 4.4.2. Libro de ventas.

Demuestra actitud positiva y responsabilidad sobre la transparencia y la rendición de cuentas. Desarrollo de ejercicios aplicando las técnicas de registro de la contabilidad de un proyecto o microempresa.

5. Libro de Balances. 6.- La Contabilidad como fuente de información para el planteamiento y toma de decisiones personales y empresariales. 6.1. Proceso contable. 6.2.Etapas del proceso contable 6.3. Cierre contable.

Análisis de los resultados del ejercicio contable de una empresa y/o de la ejecución del presupuesto personal para tomar decisiones sobre el curso de acciones a tomar.

Como el lle

Elabora informes contables de proyecto o unidad productiva escolar o comunitaria

Procedimiento sugerido para el desarrollo de la competencia No.4 Con las actividades diseñadas para el logro de esta competencia, las y los estudiantes incorporarán a sus estructuras mentales, la capacidad de conocer cómo se registran los libros contables de las diferentes formas de elaboración de los libros de contabilidad, así para poder analizar los mismos para la toma de decisiones empresariales. es una ciencia que contando con sus propios principios y normas tiene por objeto llevar cuenta y razón de las operaciones contables de una empresa o negocio. La Contabilidad

La Contabilidad información de carácter general de sobre la entidad económica estados financieros. Unapresenta presentación razonablemente adecuada la entidad se da a través mediante de las etapas del proceso contable la cual se compone del balance general, estado de resultados, el estado de utilidades retenidas y estado de flujo de efectivo. d son conceptos básicos, que establecen, delimitan e identifican en la entidad económica, las bases de cómputo de las operaciones y la presentación de la información financiera por medio de los estados financieros. Los libros de contabilidad deben estar basados en los siguientes principios generalmente aceptados: principio de entidad, según nuestro código de comercio en el artículo 14 habla sobre la personalidad jurídica. Principio de Negocio en Marcha Los principios de la contabilida

Contabilidad y Técnicas de Registros en libros de Contabilidad

Inicialmente, la o el docente, lleva a cabo una lluvia de ideas, para que las y los estudiantes personalmente, expresen los conocimientos previos que tienen con relación a: 1) ¿Qué es Contabilidad? 2) ¿Qué es un principio?

Concluida la lluvia de ideas, la o el docente explica en clase magistral los temas antes mencionados.

Contabilidad: es la ciencia que controla, registra y opera en forma sistemática toda la actividad económica de cualquier tipo de empresa. Principio: Base, srcen, razón fundamental sobre la cual se procede discurriendo en cualquier materia.(según diccionario de la real academia española) Los Principios de Contabilidad Generalmente Aceptados son los siguientes: 1. Principio de Entidad: Código de Comercio Arto. 14. Personalidad Jurídica. La sociedad mercantil constituida de acuerdo a las disposiciones de este código e inscrita en el Registro Mercantil, tendrá personalidad jurídica y propia y distinta de los socios individualmente considerados. 2. Principio de Negocio en Marcha: Código de Comercio Art. 24. El plazo de la sociedad principia desde la fecha de inscripción de la misma en el Registro Mercantil. 3. Principio de Unidad Monetaria: Código de Comercio Art. 369. Idiomas Español y Moneda Nacional. Los libros y registros deben operarse en español ylas cuentas en moneda nacional. 4. Principio del Período Contable: Código de Comercio Art. 374. Balance General y Estado de Perdidas y Ganancias. El comerciante deberá de establecer tanto al iniciar sus operaciones, como por lo menos una vez al año, la situación financiera de su empresa a través del Balance General y Estado de Pérdidas y Ganancias que deberá de ser firmado por el comerciante y el contador. 5. Principio de Costo: Ley del I.S.R. Art. 52 Para cerrar el ejercicio impositivo, el valor de las existencias de mercancías debera establecerse utilizando algunos de los siguientes métodos. 6. Principios del Conservatismo: Ley del I.S.R. Art. 38. Inciso q. Las deudas incobrables, siempre que se srcinen en operaciones del giro habitual del Negocio en que se justifique tal calificación. 7. Principio de Revelación Suficiente: Código de Comercio Art. 379 Exhibición de la Situación Financiera. El Balance General deberá expresar con veracidad y en forma razonable, la situación financiera del comerciante y los resultados de sus operaciones. 8. Principio de Consistencia o Uniformidad: Al interpretar los estados financieros de una empresa debe observarse la uniformidad de los métodos utilizados, es decir que, por ejemplo, en un método de depreciación adoptado inicialmente debe mantenerse, a la vez que se observa la necesidad de readecuación de los métodos. 9. Principio de Realización: Bajo este principio se reconocen tanto los egresos como los ingresos, los incrementos de activos derivados de las operaciones. Mide los ingresos en relación con los efectivos, etc. (9)

Autoevaluación: Las y los estudiantes se autoevalúan, con el propósito de formarse un juicio de valor acerca del nivel de aprendizaje alcanzado. El docente puede utilizar las estrategias que respondan a este propósito. Por ejemplo: Puede efectuar una Mesa Redonda para que las y los alumnos se intercambien sus conocimientos adquiridos.

Coevaluación. Para que la autoevaluación se obtenga mejores resultados, es necesario que las y los estudiantes evalúen unos con otros, el nivel de aprendizaje alcanzado durante el proceso. Se sugiere elaborar un instrumento para evaluar este proceso: Lista de cotejo, escala de rango, etc. Los indicadores deben ser claros para no confundir al estudiante.

Heteroevaluación. Es el procedimiento que realiza el docente para evaluar el nivel de aprendizaje de las y los estudiantes, considerando la auto y coevaluación. Para evaluar la actividad que realizaron el docente realiza un cuestionario a todos los estudiantes para verificar lo aprendido.

Presentación de las Diferentes Clases de Contabilidad El docente realizará equipos de trabajo de 5 a 7 alumnos entregándoles un folleto con el tema de CLASES DE CONTABILIDAD del cual realizaran un Mapa Conceptual o un Cuadro Sinóptico en papel de 80 gramos y posteriormente deberá copiarlo en su cuaderno. FOLLETO:

CLASES DE CONTABILIDAD: Según la clase de negocio de que se trate, así se llamará la contabilidad, por ejemplo: 1) En el sistema bancario se llama CONTABILIDAD BANCARIA 2) En la Industria se llama CONTABILIDAD DE COSTOS 3) En las fincas se llama CONTABILIDAD AGROPECUARIA 4) En las empresas o sociedades se llama CONTABILIDAD DE SOCIEDADES 5) En el comercio en general se llama CONTABILIDAD MERCANTIL 6) En el Estado se llama CONTABILIDAD FISCAL Y GUBERNAMENTAL

DIVISIÓN DE L A CONTABILIDAD Según la aplicación que se le dé, encontraremos que la Contabilidad puede dividirse en:

Contabilidad Comercial: Es aquella que se utiliza en los negocios de compra-venta de mercancías, por ejemplo: la que se lleva en los almacenes comerciales. Contabilidad Industrial: Es aquella que se utiliza en las empresas manufactureras, mineras, agreícolas y pecuarias, por ejemplo: la que se lleva en la industria textil. Contabilidad de Servicios: Es aquella que se utiliza en las empresas en las que la actividad de un servicio, como el transporte, los seguros, etc. Ejemplo la que se lleva en las empresas de aviación. (10)

Autoevaluación: Las y los estudiantes se autoevalúan, con el propósito de formarse un juicio de valor acerca del nivel de aprendizaje alcanzado. El docente puede utilizar las estrategias querespondan a este propósito. Por ejemplo: Puede efectuar un debate para que las y los alumnos se intercambien sus conocimientos adquiridos.

Coevaluación. Para que la autoevaluación se obtenga mejores resultados, es necesario que las y los estudiantes evalúen unos con otros, el nivel de aprendizaje alcanzado durante el proceso. Se sugiere elaborar un instrumento para evaluar este proceso: Lista de cotejo, escala de rango, etc. Los indicadores deben ser claros para no confundir al estudiante.

Heteroevaluación. Es el procedimiento que realiza el docente para evaluar el nivel de aprendizaje de las y los estudiantes, considerando la auto y coevaluación. Para evaluar la actividad que realizaron el docente realiza un cuestionario a todos los estudiantes para verificar lo aprendido

Presentación de los Libros Obligatorios y Auxiliares de Contabilidad.

El docente aplicará una tarea de investigación sobre los Libros Obligatorios y Auxiliares y posteriormente les realizará preguntas directas a los alumnos respecto a la investigación Preguntas sugeridas para realizar a los alumnos: 1) Cuántos libros de Contabilidad son? 2) Cuáles son los libros de Contabilidad? 3) Cuáles son los libros Obligatorios? 4) Cuáles son los libros Auxiliares? 5) Porqué algunos libros de Contabilidad son Obligatorios? 6) Porqué algunos libros de Contabilidad son Auxiliares? 7) Qué es un Inventario? 8) Qué es el Libro Diario?

LIBROS OBLIGATORIOS DE CONTABILIDAD Los libros de Contabilidad según el Código de Comercio de Guatemala Decreto 2-70 del Congreso de la República, en su artículo 368 establece que todos los comerciante deben llevar los siguientes libros: Inventario, Diario (o de Primera Entrada), Mayor (o Centralizador) y el Libro de Balances (o de Estados Financieros). Pero también hay que agregar que la ley del IVA Decreto 27-92 establece como también obligatorios los libros de Compras y Ventas. Estos libros se les llaman obligatorios porque existe una ley que los exige como tal e instituciones de Estado que velan porque así sea, tales como la SAT y el Registro General Mercantil, entre otras. Los Libros anteriores deben ser Habilitados por la Superintendencia de Administración Tributaria y Autorizados por el Registro General Mercantil de Guatemala, tal como lo exige el artículo 372 del Código de Comercio de Guatemala. Exceptuando los libros de Compras y ventas que solo deben ser habilitados por la SAT. La misma ley en su artículo 373 establece claramente las operaciones, errores u omisiones y el procedimiento que debe realizarse contablemente en estos casos específicos.

LIBRO DE INVENTARIOS: Es un libro principal y obligatorio que se utiliza contablemente para registrar todos losbienes, derechos, cargas u obligaciones y el patrimonio neto que un ente económicamente activo posee. Consta de dos columnas el libro formal, sus columnas se llaman subtotal y Total, pero de igual forma algunos contadores le llaman Debe y Haber, pero su procedimiento y resultado debe estar Basado según las Normas Internaciones de Contabilidad, NIC. También se puede trabajar en tres columnas pero se sugiere trabar en Dos por formalidades Contables. LIBRO DE DIARIO: Es un libro principal y obligatorio, que también recibe el nombre De primera Entrada. Se utiliza para registrar todos los movimientos y todas las transacciones comerciales de un Ente económicamente activo. En el se realizan cronológicamente partidas simples, compuestas o mixtas según la actividad que económicamente que se este ejecutando. Se hacen uso de las cuentas según su naturaleza contable, de activo, pasivo, pérdida o ganancia, en las cuales se cargan o se abonan según los principios de partida Doble. LIBRO MAYOR: Es un libro principal y obligatorio, que también recibe el nombre de Centralizador, y se le llama así porque en el debe de Registrarse todas las cuentas provenientes del libro Diario, para operarlas y trasladar los saldos de cada cuenta al libro de Balances. Este libro se trabaja formalmente a Doble Folio, pero también se puede trabajar en “T” graficas o incluso en tres columnas, según el criterio contable que se utilice. LIBRO DE BALANCES: Es un libro principal y obligatorio, que también recibe el nombre De Estados Financieros. En el se registra el Balance General de Apertura, los Balances de Saldos ordinarios y extraordinarios, y los Estados Financieros ( estado de Resultado y Balance General). Se le llama de Saldos porque allí se registran todos los saldos de las cuentas utilizadas en nuestra contabilidad. Se trabaja formalmente en un libro de cuatro columnas y se puede manejar debe, haber, Deudor y Acreedor, pero también dependerá del criterio contable que se maneje siempre y cuando estén dentro de los principios y normas de contabilidad generalmente aceptados.

LIBROS AUXILIARES DE CONTABILIDAD. De conformidad con el Código de Comercio deGuatemala Decreto 2-70 del Congreso de laRepública, en su artículo 368 establece que los Comerciantes podrán utilizar los otros libros que se estimen necesarios por exigencias contables o administrativas o en virtud de otras leyes especiales. Estos libros no son de carácter obligatorios pero, la ley permite llevarlos para facilitar un mejor control contable-administrativos en las empresas entre ellos podemos citar: Libro de Caja, de Caja Chica, Libro de Cuentas por Cobrar (Libro de Clientes y de Deudores) Libro de Cuentas por Pagar (Libro de Proveedores y de Acreedores) libro de Salarios (Sueldos), libro de Bancos, hoja de trabajo, Libro de Actas, otros.

LIBRO DE CAJA. Es un libro principal y Auxiliar para un ente económicamente activo que esta basado su existencia en ley, pero que no tiene característica de obligatorio, más si es de suma importancia porque a las empresas les sirven para controlar sus ingresos y egresos que ejecutan en forma efectiva. Este libro se trabaja formalmente en dos columnas, pero también se puede trabajar en tres columnas según el criterio contable que se utilice. LIBRO DE BANCOS. Es un libro principal y Auxiliar para un ente económicamente activo que esta basado su existencia en ley, pero que no tiene característica de obligatorio, más si es de suma importancia porque a las empresas, les sirven para controlar sus cuentas bancarias, depósitos y retiros de fondos económicos monetarios o de ahorros, controlados a través del sistema financiero de cheques. Este libro debe mantenerse al día siempre porque su saldo debe coincidir con el Saldo de lacuenta en el banco y con elSaldo de la Cuenta Bancos en el Balance. LIBRO DE SALARIOS. Este es un libro auxiliar que se utiliza para llevar el control de las planillas o nominas de sueldos de cada uno de los trabajadores, el puesto, salario que devengan, fecha deinicio de labores entre otros. Este libro es importantísimo para una empresa ya que le permite controlar a todos sus trabajadores y que a la hora de producirse el pago de prestaciones laborales se sabe exactamente cuando empezó a laborar, con que salario inicio y con que salario finalizo su relación laboral entreel patrono y el trabajador. LIBRO DE CLIENTES: Este es un libro auxiliar que se utiliza para llevar el control de las personas o empresas que le debe a nuestra Empresa por mercadería otorgada al riguroso crédito. En el se consigna los datos cualitativos y cuantitativos de cada cliente, nombre, descripción del producto, tiempo acordado de pago etc. LIBRO DE PROVEEDORES: Este es un libro auxiliar que se utiliza para llevar el control de las personas o empresas que les debemos como Empresa por mercadería otorgada al riguroso crédito. En el se consigna los datos cualitativos y cuantitativos de cada proveedor que le debemos, en el se consigna nombre, descripción del producto, tiempo acordado de pago etc. LIBRO AUXILIAR HOJA DE TRABAJO: Este es un libro auxiliar que se utiliza para facilitar la presentación de los Estados Financieros de un ente económicamente activo. Se puede trabajar en 8, 10, 12, 14, 16, 18 columnas según el criterio contable que se maneje y el tipo de contabilidad. (11)

Heteroevaluación. Es el procedimiento que realiza el docente para evaluar el nivel de aprendizaje de las y los estudiantes, considerando la auto y coevaluación. Para evaluar la actividad que realizaron el docente realiza un cuestionario a todos los estudiantes para verificar lo aprendido

Aplicación de la Nomenclatura de cuentas. El docente realizará 4 equipos de trabajo de 6 a 8 alumnos y le entregará un folleto con la nomenclatura de cuentas a cada equipo de trabajo. Al equipo No 1 le asignará las cuentas de Activo, el equipo No 2 las cuentas de Pasivo, al equipo No 3 las cuentas de Pérdidas y al equipo No 4 las cuentas de Ganancia. Cada grupo copiará las respectivas cuentas en papel de 80 gramos a fin de realizar carteles y posteriormente copiarán todas las cuentas a su cuaderno.

NOMENCLATURA DE CUENTAS 1.

ACTIVO Es el total de bienes materiales e inmateriales estimados en dinero que son propiedad de la empresa. Se divide en: Activo Corriente, Activo No Corriente y Otros Activos No Corrientes.

1.1

ACTIVO CORRIENTE Es el que agrupa todas las cuentas cuyo saldo varía constantemente, es decir: que aumenta o disminuye conforme las operaciones que se realicen.

1.1.1 CAJA Registra el dinero en efectivo. Incluyendo los cheques que se reciben en pago. 1.1.2 CAJA CHICA Registra el dinero en efectivo que se destina para el pago de gastos pequeños. 1.1.3 BANCOS Representa el dinero depositado en losbancos. Debe anotarse el nombrede cada banco. 1.1.4 MONEDA EXTRANJERA Representa las divisas (monedas) deotros países que se tengan en existencia. 1.1.5 CLIENTES Comprende aquellas personas que nos deben el valor de mercaderías que les dimos al crédito es decir, sin que nos firmen ningún documento. 1.1.6 DOCUMENTOS O EFECTOS POR COBRAR Representa el valor de todos los documentos que terceras personas han firmado a favor de nuestra empresa 1.1.7 DEUDORES DIVERSOS Registra el dinero que algunas personas nos deben por cualquier motivo, siempre que no sea por mercaderías y que no nos hayan firmado ningún documento. 1.1.8 MERCADERIAS Registra los artículos que se han adquirido con el fin de obtener una ganancia en su venta. 1.1.9 ACCIONES O VALOR MOBILIARIOS Comprende el valor de los documentos que respaldan una inversión de nuestra empresa en otra.

1.1.10 CUENTAS POR COBRAR Registra el dinero que nos deben algunas personas, el cual dimos en calidad de préstamo sin respaldo de documentos a nuestro favor. 1.1.11 ESPECIES FISCALES Registra el valor de papel sellado, timbres fiscales que se tengan para uso de la empresa. 1.1.12 CREDITO FISCAL Impuesto que en una empresa se ha pagado al comprar mercaderías. 1.1.13 CUENTA PROMESA Es conocida también con el nombre Cuenta de Aportación, esta cuenta comprende la parte comprometida por un socio ante la sociedad, aún cuando no la haya hecho efectiva al momento de la apertura contable. 1.1.14 CUENTA PARTICULAR O CUENTA PERSONAL Esta cuenta puede ser igualmente deudora o acreedora. Es deudora cuando la sociedad ha entregado en calidad de préstamo cierta cantidad o valores, al socio; por lo tanto se adquiere el derecho de recuperarlo. Es acreedora, en el caso contrario, cuando el socio ha entregado a la sociedad bienes o dinero en calidad de préstamo, por lo tanto, en este caso, es una obligación de la sociedad con el socio. 1.1.15 CUENTA ASIGNACION Representa cantidades de dinero que el socio recibe a cuenta de posibles utilidades futuras, es cuenta de saldo deudor y se cierra al final del ejercicio con cargo a la cuenta beneficios

1.2

ACTIVO NO CORRIENTE: Agrupa todas las cuentas que representan valores o bienes de uso permanente en la empresa.

1.2.1 MOBILIARIO Y EQUIPO Corresponde a esta cuenta todos los muebles y aparatos de oficina que estén en servicio. 1.2.2 TERRENOS Es el valor de los espacios de tierras que posee la empresa. 1.2.3 EDIFICIOS Representa el valor de las casas, edificios y locales comerciales propiedad del comerciante. 1.2.4 VEHÍCULOS Registra el valor de cualquier aparato destinado al transporte de cosas o de personas, siempre que sean de uso para la empresa. 1.2.5 INMUEBLES Son todos los terrenos, casas, etc. Que son propiedad de la empresa, estimados en dinero. 1.2.6 MAQUINARIA Registra el valor de la maquinaria propiedad de la empresa y que este en servicio. 1.2.7 HERRAMIENTAS Registra el valor de la herramienta que se tenga al servicio de la empresa. 1.2.8 EQUIPO DE COMPUTACION Registra el valor de los bienes propiedad de la empresa que se relacionan con el equipo de computación, en sí misma las computadoras.

1.3

OTROS ACTIVOS NO CORRIENTES Comprende todas las cuentas que registran valores que se han pagado en forma anticipada, representan un derecho para la empresa y por lo tanto son un activo.

1.3.1 GASTOS DE ORGANIZACIÓN Son todos los gastos efectuados por laorganización. Legalización y funcionamiento de laempresa. 1.3.2 GASTOS DE INSTALACION Son todos los gastos efectuados por las instalaciones hechas en el edificio para la empresa. 1.3.3 DERECHO DE LLAVE Registra el monto que se cancela por un local o edificio, que ya se encuentra acreditado comercialmente, en el cual se desea poner otro negocio. 1.3.4 SEGUROS PAGADOS POR ANTICIPADO Es el valor pagado anticipadamente para asegurar los muebles, vehículos, mercaderías. Contra robos, incendios, etc. 1.3.5 PAPELERIA Y UTILES DE OFICINA POR ANTICIPADO Registra el valor del material y demás accesorios de oficina para uso del negocio siempre que se compren en regulares cantidades. 1.3.6 ALQUILERES PAGADOS POR ANTICIPADO Incluye todos los alquileres que se hagan por adelantado. 1.3.7 ADELANTOS AL PERSONAL O SUELDOS PAGADOS POR ANTICIPADO Registra el valor que los trabajadores cobren en forma anticipada a cuenta de su sueldo. 1.3.8 INTERESES PAGADOS POR ANTICIPADO Registra intereses que se paguen en forma anticipada, por préstamos que se hayan recibido. 1.3.9 COMISIONES PAGADAS POR ANTICIPADO Representa cualquier dinero pagado en concepto de comisiones de cualquier naturaleza siempre que tengan relación con la actividad comercial de la empresa. 1.3.10 PROPAGANDA O PUBLICIDAD PAGADA POR ANTICIPADO Debe usarse uno de los dos nombres, para operar en libros el pago de publicaciones de cualquier tipo que se hayan pagado por anticipado. 2.

PASIVO

Es el total de deudas u obligaciones que la empresa debe cancelar a corto o largo plazo. Se considera corto plazo a las deudas que debe liquidarse en un tiempo máximo de un año, se considera largo plazo a las deudas que serán canceladas de un año en adelante. Hay tres principales clases de pasivo: Corriente, No Corriente y Otros Pasivos No corrientes.

2.1

PASIVO CORRIENTE Agrupa todas las cuentas que representan deudas que han de cancelarse a corto plazo, es decir, a un tiempo máximo de un año.

2.1.1 DOCUMENTOS POR PAGAR Cuenta que comprende los documentos comerciales y de crédito que la empresa o su propietario firmen a favor de terceras personas o empresas con vencimiento a corto plazo. 2.1.2 CUENTAS POR PAGAR Representa el dinero que debemos a cualquier persona o empresa que nos dieran en préstamo, sin haber firmado ningún documento.

2.1.3 PROVEEDORES Representa a las personas o empresas a quienes les compramos mercaderías al estricto crédito, es decir: sin firmar ningún documento. 2.1.4 ACREEDORES Comprende a las personas o empresas a quienes debemos por cualquier motivo distinto al anterior y siempre que no hayamos suscrito ningún documento. 2.1.5 SUELDOS Y SALARIOS POR PAGAR Registra los sueldos de los trabajadores que no se hayan cancelado. 2.1.6 IMPUESTOS VARIOS POR PAGAR Registra todo impuesto que por alguna razón no se canceló en el plazo asignado por determina ley tributaría. 2.1.7 RETENCIONES IGSS Cuenta que registra el dinero que descontamos a los trabajadores para el seguro social, el cual aún no se ha pagado al IGSS. 2.1.8 CUOTA PATRONAL IGSS POR PAGAR Valor de las cuotas que el patrono adeuda al IGSS, por concepto de seguro social . 2.1.9 DEBITO FISCAL Cuenta que registra el valor del impuesto que la empresa ha cobrado en sus ventas.

2.2

PASIVO NO CORRIENTE Agrupa todas las cuentas que representan deudas y obligaciones que se contraen Para cancelarlas a largo plazo, esdecir mayor de un año.

2.2.1 HIPOTECAS O ACREEDORES HIPOTECARIOS Registra el valor de los préstamos que recibimos de otras empresas o entidades de crédito, por los que dejamos como garantía los documentos de propiedad de un bien inmueble, terreno, etc. Siempre que su fecha de vencimiento se a de largo plazo. 2.2.2 DOCUMENTOS O EFECTOS POR PAGAR esta cuenta representa el valor total de los documentos pendientes de pago, tales como letras de cambio, pagarés, etc., siempre que la fecha de vencimiento sea a largo plazo.

2.3

OTROS PASIVOS NO CORRIENTES Son todos los cobros hechos por anticipado, a cuenta de servicios que todavía no hemos prestado y que por lo tanto se consideran como unadeuda. También se le llama créditos diferidos.

2.3.1 COMISIONES COBRADAS POR ANTICIPADO Comprende lo que se cobra por anticipado en concepto de servicios a prestar. 2.3.2 ALQUILERES COBRADOS POR ANTICIPADO Registra el valor de los alquileres que se cobran en forma adelantada. 2.3.3 INTERESES COBRADOS POR ANTICIPADO Registra el valor que nos pagan por anticipado por concepto de algún préstamo que hemos concedido.

2.4

PASIVO DE REGULARIZACIÓN DE ACTIVO Registra las cuentas que se rebajan del activo en el estado de resultados.

2.4.1 RESERVA PARA DEPRECIACIONES Registra todos los valores porcentuales que se calculan del activo fijo de acuerdo a la ley del impuesto sobre la renta, también se le denomina DEPRECIAICONES ACUMULADAS

2.4.2 RESERVA PARA AMORTIZACIONES Registra todos los valores que se calculan del activo diferido de acuerdo a la ley del impuesto sobre la renta, también se le denomina AMORTIZACIONES ACUMULADAS

2.5

PASIVO EVENTUAL Registra cualquier provisión que sehaga para cuenta de beneficios futuros.

2.5.1 RESERVA PARA GRATIFICACIONES Provisión que se hace por posibles compensaciones o gratificaciones que se presenten durante el período contable. 2.5.2 RESERVA PARA JUBILACIONES Comprende el dinero que se aparta o se reserva para hacer frente a las jubilaciones que s efectúan durante un ejercicio contable. 2.5.3 RESERVA PARA INDEMNIZACIONES Provisión que se efectúa para poder pagar a determinados trabajadores, dicha prestación laboral que consiste en el pago de un sueldo mensual por año de trabajo. 3. CAPITAL Valor permanente de lo que ocasiona rentas, intereses o utilidades. Valor del patrimonio. 3.1

CUENTA CAPITAL O CAPITAL LÍQUIDO Registra la diferencia entre el total del activo menos el total del pasivo, y representa el valor del patrimonio líquido, propiedad del empresario o comerciante.

3.2

GANANCIA NETA DEL EJERCICIO Cuenta con la que se registra la utilidad o ganancia líquida obtenida al final del período contable.

3.3

PÉRDIDA NETA DEL EJERCICIO Registra el valor de la pérdida líquida de determinado período contable.

3.4

CUENTA CAPITAL Es el monto de las aportaciones totales de los socios, mediante la ya conocida ecuación patrimonial: Activo menos Pasivo igual Capital. El cual es conocido como CAPITAL SOCIAL y no puede ser alterado por la pérdida y la ganancia.

3.5

CAPITAL NETO Es el resultado de la aplicación final de la pérdida o la ganancia de un período contable.

3.6

CAPITAL COLECTIVO Suma total de los capitales aportados.

3.7

CAPITAL ACCIONADO Es el que está dividido por Acciones. (Capital Autorizado)

3.8

SOCIAL INDIVIDUAL Es el capital aportado por cada socio individualmente.

CUENTAS DE RESULTADOS (pérdidas y ganancias): Son aquellas que se registran ya sea una ganancia, o una pérdida en el curso de las actividades de la empresa. A estas cuentas se les conoce también con el nombre de Cuentas de Gastos y Productos. Para comprender mejor el papel que desempeñan estas cuentas, recordemos que cuando se dice que en una operación hubo ganancias, se da a entender que recibimos un valor, sin dar ni quedar a deber nada en cambio, o que si dimos algo fue a aun precio superior al de adquisición. Cuando se dice que en una operación hubo perdida, se da a entender que dimos un valor, sin recibir y sin que nos hayan quedado a deber nada a cambio, o que si recibimos algo fue a un precio inferior al de la cosa entregada. A continuación se le presentan algunas cuentas que indican pérdidas y ganancias.

PERDIDA: Intereses pagados: Es el precio pagado por cierta deuda adquirida en los bancos o por el no pagar a tiempo cierta deuda. (Intereses de préstamos) Comisiones pagadas: Representa lo que se paga por determinado servicios prestados. Descuentos concedidos: son los descuentos que la empresa le concede a sus clientes por el pronto pago de una deuda. Alquileres pagados: Representa el valor de los arrendamientos de locales u otro bien que se este arrendando. Sueldos, salarios y honorarios: Esta cuenta registra lo que ganan mensualmente los empleados de las oficinas de la empresa los cuales pueden ser de OFICINA Y SALA DE VENTA. Gastos de venta: Comprende todo gasto relacionado directamente con el despacho de la mercadería vendida, tales como fletes que se pagan para transportar mercancía vendida. Depreciaciones: Es la cuenta que se usa para registrar la disminución de valor que sufre un objeto material. En otras palabras, esta cuenta representa el desgaste material que sufren ciertos activos fijos tangibles. (MOBILIARIO Y EQUIPO, VEHICULOS ETC.) Compras: En esta cuenta figura el valor de la mercadería adquirida en el curso del periodo contable, si es que se quiere separarla de la mercadería que ya se tenía al empezar tal periodo. Gastos de compras: En esta cuenta comprende todos los gastos concernientes con la adquisición de las mercaderías, pero que no entran bajo la cuenta “Compras”; tales como el pago de transportar la mercadería comprada. Gastos particulares: También conocido por gastos personales, comprende toda la cantidad que retire el propietario para atender sus gastos privados. Gastos de administración o Administrativos: Comprende a los sueldos del gerente, sub-gerente, administrador, agente, contador y auditor de la empresa, así como gastos de representaciones y viáticos de los mismos. Descuento sobre venta: Representa al descuento que se aplica para realizar una determinada venta con más rapidez. Devoluciones sobre venta: Representa la devolución de mercadería que hemos vendido anteriormente. GANANCIA: Interese percibidos: son los interese que cobramos por prestar dinero o por algún documento que contenga una tasa interés incluida. Comisiones percibidas: Representa lo que se cobra por determinados servicios prestados. Descuentos percibidos: Son los descuentos que se aplican a ciertos pagos que se realizan antes del plazo correspondiente. Alquileres cobrados: Representa el valor cobrado por arrendamiento de ciertos bienes. Dividendos: Representa la ganancias que paga la empresa a sus accionistas. Ventas: En esta cuenta figura el valor de la mercadería vendida durante el periodo contable. Devoluciones sobre compras: Representa el valor de las mercaderías que son devueltas al proveedor. Descuento sobre compra: Descuento que los proveedores nos hacen por la compra hecha o por pronto pago.

El gran problema con que el alumnado se encuentra es que le cuesta distinguir o identificar si las cuentas pertenecen al balance (ACTIVO, PASIVO), o si pertenecen a cuentas de Resultados (PÉRDIDAS Y GANANCIAS), ya que las palabras por cobrar y por pagar al igual que las palabras cobrados y pagados muchas veces da lugar a confusión. Por lo que se le recomienda al catedrático hacer énfasis en lo siguiente: Se debe recordar siempre lo siguiente: Todo lo “Por cobrar” es ACTIVO: O sea que nosotros (la empresa) va a cobrar deudas en un futuro próximo y no como a veces erróneamente se piensa, que la empresa se lo van a cobrar. Todo lo “Por Pagar” es PASIVO: O sea que nosotros (la empresa) va a pagar deudas en un futuro próximo y no como erróneamente se piensa, que a la empresa se lo van a pagar. Todo lo “Pagado” es PERDIDA: Esto es que nosotros como empresa hemos cance lado o pagado ciertos gastos y no como equivocadamente se piensa, que a nosotros nos han pagado. Todo lo “Cobrado” es GANANCIA: Esto es que nosotros como empresa hemos cobrado o nos han pagado, y no como equivocadamente se piensa, que la empresa le han cobrado o ésta ha pagado.(12)

Autoevaluación: Las y los estudiantes se autoevalúan, con el propósito de formarse un juicio de valor acerca del nivel de aprendizaje alcanzado. El docente puede utilizar las estrategias querespondan a este propósito. Por ejemplo: Puede efectuar una mesa redonda para que las y los alumnos se intercambien sus conocimientos adquiridos.

Coevaluación. Para que la autoevaluación se obtenga mejores resultados, es necesario que las y los estudiantes evalúen unos con otros, el nivel de aprendizaje alcanzado durante el proceso. Se sugiere elaborar un instrumento para evaluar este proceso: Lista de cotejo, escala de rango, etc. Los indicadores deben ser claros para no confundir al estudiante.

Heteroevaluación. Es el procedimiento realiza el docente evaluar el nivel de y los realiza estudiantes, considerando la autoque y coevaluación. Parapara evaluar la actividad queaprendizaje realizaron de el las docente un cuestionario a todos los estudiantes para verificar lo aprendido

Apertura de la Contabilidad Libro Inventario

El docente presenta a los estudiantes la siguiente información y posteriormente el ejercicio para resolverlo en clase para que ellos tengan los lineamientos a seguir para la elaboración de un libro Inventario.

LIBRO DE INVENTARIOS: Es un libro principal y obligatorio que se utiliza contablemente para registrar todos los bienes, derechos, u obligaciones el patrimonio neto queseunllaman ente económicamente posee. Consta de cargas dos columnas el libroy formal, sus columnas subtotal y Total, activo pero de igual forma algunos contadores le llaman Debe y Haber, pero su procedimiento y resultado debe estar Basado según las Normas Internaciones de Contabilidad, NIC. También se puede trabajar en tres columnas pero se sugiere trabar en Dos por formalidades Contables.

Partes de que consta el I nventario: Encabezado: Comprende el nombre Inventario, Número de Inventario, Nombre del negocio, Nombre del propietario y la fecha a la que se esta practicando el inventario. Cuerpo: Es la parte medular del inventario, esta parte esta integrada por los valores de Activo, Pasivo y Capital, Patrimonio Neto.

Resumen: Comprende el listado de cuentas de Activo, Pasivo y Capital que integran el inventario. Certificación del Inventario: Después del resumen se hace una certificación del capital o patrimonio neto de la empresa, la cual se expresa en letras y números. Fecha: Se escribe la fecha en que se esta haciendo el inventario, esta generalmente es igual a la que se consigna en el encabezado. Firmas: El inventario lleva las firmas del Contador y del propietario o representante legal de la entidad.(13)

EJEMPLO DE LIBRO INVENTARIO REALIZAR EL SIGUIENTE IINVENTARIO DE LA EMPRESA LOS PIWIS PROPIEDAD DEL SEÑOR CARLOS EDUARDO GARCÍA GARCÍA, AL 01 DE ENERO DE 2011. PRESENTANDOLE LA SIGUIENTE INFORMACIÓN:

Artículos para la venta: 20 Televisores marca Sony Q2,240.00 c/u con iva incluido. 10 Licuadoras Oster Q 168.00 c/u con iva incluido. 5 Refrigeradoras Oster Q 5,600.00 c/u con iva incluido.

De los artículos para la venta mencionados anteriormente se le adeudan a las siguientes empresas lo siguiente: Agencias Way Q10,000.00 Tiendas La Curacao Q 7,500.00 El Gran Gallo Q5,000.00 Distribuidora el Esfuerzo Q 5,000.00 Distribuidora La Mascota Q 4,000.00

Se adeudan los siguientes documentos (La empresa le debe a otras empresas). 1 pagaré a favor de Toyota S.A. a 2 años Q10,000.00 1 Letra de Cambio a Favor de Honda S.A. a 13 meses Q5,000.00

Nos adeudan las siguientes personas por mercadería: Pedro Guerra Q5, 000.00 Carlos Paz Q2, 000.00

Nos adeudan los siguientes documentos (Nos deben) 1 pagaré de Juan González a 2 años Q 2,500.00 1 Letra de Cambio de Carlos Hernández a 3 años Q 2,500.00

La empresa cuenta con los siguientes Bienes: 1 pick-up marca Mazda color negro placas No. Ck121 chassis no. 23233 motor k6766 Q56,000.00 c/iva incluido. 1 Motocicleta marca Honda color negro placas C232 chassis no. 333 motor 110 Q11,200.00 c/iva incluido. 1 Motocicleta Marca Italika color roja placas C222 chassis no. 637 motor 125 Q 14,000.00 c/iva incluido.

2 Mostradores de madera 5 Estanterías de Metal

Q1,680.00 C/uno C/iva incluido. Q 1,120.00 c/uno C/iva incluido.

Monedas Q1,000.00 Billetes Q 10,000.00 Cuenta Monetaria en el Banco Agromercantil cuenta No. 323232123 Q10,000.00 Cuenta Monetaria en el Banco de Antigua cuenta No. 43434343 Q10,000.00

Inventario No. 1 De la empresa Los Piwis propiedad del señor Carlos Eduardo García García, al 01 de enero de 2011.(Cifras expresadas en Quetzales). ACTIVO NO CORRIENTE VEHICULOS

1

Pick- up marca Mazda color negro placas No. CK121 chassis no. 23233 motor k6766 Q56,000.00 c/iva

1

chassis no. 333 motor 110

1

Q 50,000.00

Q 50,000.00

Q 10,000.00

Q 10,000.00

Q 14,000.00

Q 14,000.00

Motocicleta marca Honda color negro placas C 232 Q 11,2000.00 c/iva

Motocicleta marca Italika color roja placas C222 chassis no. 637 motor 125

Q 14,000.00 c/iva

Q 74,000.00

MOBILIARIO Y EQUIPO

2

Mostradores de madera

Q 1,680.00 c/uno C/iva

Q 1,500.00

Q

3,000.00

5

Estantería de metal

Q 1,120.00 c/uno C/iva

Q 1,000.00

Q

5,000.00

Q

8,000.00

Q

5,000.00

DOCUMENTOS POR COBRAR LARGO PLAZO

1

Pagaré de Juan González a 2 años

Q 2,500.00

Q

2,500.00

1

Letra de Cambio de Carlos Hernández a 3 años

Q 2,500.00

Q

2,500.00

CORRIENTE CAJA Billetes

Q 10,000.00

Monedas

Q

1,000.00

Q 11,000.00

BANCOS Cta Monetaria Banco agromercantil # 323232123

Q 10,000.00

Cta Monetaria Banco de Antigua # 43434343

Q 10,000.00

Q 20,000.00

CLIENTES Pedro Guerra

Q

5,000.00

Carlos Paz

Q

2,000.00

Q

7,000.00

MERCADERIA

20

Televisores marca Sony

10

Licuadoras Oster

Q2,240.00 C/U C/Iva Q 168.00 C/U C/Iva

Q 2,000.00 Q

5

Refrigeradoras Oster

Q5,600.00 C/U C/Iva

Q 5,000.00

150.00

Q 40,000.00 Q

1,500.00

Q 25,000.00

Q 66,500.00

IVA POR COBRAR

12%

Sobre mercadería, Mobiliario y equipo, y Vehículos Q148,500.00

Q 17,820.00

Total Activo

Q 209,320.00

PASIVO NO CORRIENTE DOCUMENTOS PORPAGAR LARGO PLAZO

1

Pagaré a favor de Toyota S.A.

Q 10,000.00

Q 10,000.00

1

Letra de Cambio a favor de Honda S.A.

Q 5,000.00

Q

Agencias Way

Q 10,000.00

Q 10,000.00

Tiendas La Curacao

Q 7,500.00

Q

7,500.00

El Gran Gallo

Q 5,000.00

Q

5,000.00

5,000.00

Q 15,000.00

CORRIENTE PROVEEDORES

VAN

Q 22,500.00

Q 15,000.00

VIENEN

Q 22,500.00

Distribuidora El Esfuerzo

Q

5,000.00

Distribuidora La Mascota

Q

4,000.00

Total Pasivo

Q 15,000.00

Q 31,500.00 Q 46,500.00

PATRIMONIO NETO Cuenta Capital señor García

Q 162,820.00

Suma Igual al Activo

Q 209,320.00

RESUMEN ACTIVO Vehículos

Q 74,000.00

Mobiliario y equipo

Q

Documentos por cobrar largo plazo Caja

Q 5,000.00 Q 11,000.00

8,000.00

Bancos

Q 20,000.00

Clientes

Q

MERCADERIA

Q 66,500.00

Iva por cobrar

Q 17,820.00

7,000.00

PASIVO Documentos por pagar largo plazo

Q 15,000.00

Proveedores

Q 31,500.00

Cuenta Capital señor García

Q 162,820.00

Sumas iguales

Q 209,320.00

Q 209,320.00

El infrascrito Perito Contador registrado bajo el # 110,510, ante la Superintendencia de Administración Tributaria (SAT) CERTIFICA: que el capital de la empresa Los Piwis propiedad del señor Carlos Eduardo García García Responsabilizándome asciende a Doscientos nueve mil trescientos veinte quetzales (Q209,320.00). por cualquier inexactitud que contenga el mismo. con cero centavos

San Juan Bautista, Suchitepéquez o1 de Enero 2011

P.C. __________________________ Soyla Luz de la Esperanza Arango

Propietario__________________________ Carlos Eduardo García García

Ejercicios de Inventarios Propuestos LABORATORIO # 1 ELAB ORAR UN INVE NTARIO : El 01 de enero del presente año, el señor Casimiro Buena Vista, propietario del Almacén “Soy Bueno”, que se dedica ala compra y venta de camas, inicia sus operaciones mercantiles, por lo que le solicita a usted le elabore su inventario, respectivo resumen y certificación del capital. Proporcionándole para el efecto la documentación necesaria de donde usted extrae la siguiente información. Utilice hojas tabulares de 3 columnas. DATOS PARA

Efectivo: billetes Q.2,670.00, Monedas Q.240.00, Cheques: No. 3210 del BAM S.A. Q.2,000.00, No. 1400 del Banco Banrural S.A. por Q.1,100.00. No. 4-87655-4 del Banco Industrial S.A por Q.69,540.00. G & T Continental S.A. por Q.47, 231.00. Cuentas Bancarias:

No. 3-12345-2 del Banco

IVA: De IVA pagó el 12% sobre los bienes afectos a éste impuestos, como: las mercaderías, mobiliario y

equipo, equipo de computación, los libros de contabilidad y la papelería y útiles. En dichos valores el IVA ya está incluido. Prestamos concedido: Un recibo No. 3 de Panfilo Picón por un préstamo que se concedió de Q.1,000.00. Ar tícu lo s p ara la v enta: (con el IVA incluido) SABANAS:

100 King Size, c/u. Q.56.00; 100 Matrimonial c/u Q.44.80; 100 Semimatrimonial, c/u. Q.33.60; 100 imperial, c/u. Q.22.40. COLCHAS: 100 King Size, c/u. Q89.60; 100 Matrimonial, c/u. Q.78.40; 100 Semimatrimonial, c/u. Q.67.20; 100 Imperial, c/u. Q.56.00; 100 Small, c/u. Q.44.80. De esta mercadería de adeuda a: distribuidora La Momosteca Q.10,000.00 y Almacén las Chamarras Q.8,000.00. Mobiliario y E

quip o: (con el IVA incluido) 5 estanterías de metal de 5 compartimientos, c/u. Q.784.00; 3 mostradores de madera con vidrios, c/u. Q.1,008.00; 1 escritorio ejecutivo de madera de 5 gavetas, Q,2,016.00; 1 escritorio secretarial de madera de 3 gavetas Q.952.00; 1 silla ejecutiva giratoria de metal y esponja Q. 1,456.00; 1 Silla secretarial. 16,016.00; 1 Monitor marca Epson, Q.6496.00; 1 Impresora marca Epson, Q.9,632.00. De éste equipo se adeuda a Microsys, S.A. 10 letras de Q.1,000.00 c/u. Gastos : 6 libros de contabilidad, c/u. Q35.84 (Con el IVA incluido); 1 Recibo No.6253 de Registro Mercantil por inscripción y comerciante y empresa Q.175.00; 1 Recibo SAT No. 52-74386 por Habilitación de los libros de contabilidad, Q.300.00; 1 Recibo No. 3490 del registro Mercantil por Autorización de libros, Q.60.00; 1 timbre fiscal para la patente de comercio, Q.50.00. Libros d e contabil idad y otros

Pap ele ría y Útil es : (Con el IVA incluido) 5 resmas de papel bond de 60 gramos tamaño carta, Q.26.88

c/u.; 1 gruesa de lapicero Tec Q.144.48; 1 gruesa de Lápices Venus, Q.96.32; Una caja de sobres tamaño oficio Q.14.56.

LABORATORIO # 2 El 01 del presente mes el Sr. Napoleón Panza Blanca, inicia su negocio en compra y venta de telas, denominado Almacén “El Catrín, por lo que le solicita a usted que le elabore el inventario, resumen y certificación del capital. Utilice Hojas tabulares de 3 columnas. Para el efecto le presenta la siguiente información. (Las cuentas no están en orden). ”

IVA: ya esta incluido en los bienes muebles para el servicio de la empresa, los libros de contabilidad, los

productos para la venta y renta del local.

(Con IVA incluido) 4 Estantes de metal de 5 compartimientos, c/u. Q.716.80; 4 Mostradores de madera, c/u. Q.772.80. 2 Archivos de metal de 4 gavetas c/u. Q.907.20. 2 Máquinas de escribir eléctricas, marca Brother, c/u. Q.3,236.80. Una caja registradora marca Casio Q.5,835.20. 2 Mesas para las máquinas de escribir, c/u. Q392.00. Una mesa para la caja registradora, Q.537.60. Muebles para uso del negocio:

Muebles y Equipos S.A Q.5,000.00 sin haber firmado ningún documento. (Utilice la cuenta Acreedores No Comerciales y regístrelo en pasivo Corriente. De estos bienes se adeuda a:

6 Libros de contabilidad cada uno a Q.31.92 (Con el IVA incluido). 1 Recibo No. 6566 del Registro Mercantil Por Inscripción de comerciante y empresa, Q.175.00. 1 Timbre fiscal para la patente de comercio, Q.50.00. 1 Recibo SAT-0052-8312 por habilitación de los libros Q.300.00. 1 Recibo No.6567 del Registro Mercantil por autorización delos libros Q.60.00. Libros d e contabil idad y otros Gastos:

Efectivo: Billetes Q3240.00. Monedas Q.240.00. Cheques: No. 34 Banco de Desarrollo rural S.A. Q.2,000.00. No. 54 Banco Agromercantil S.A. Q.1,400.00. la venta: (Con el IVA incluido) 10 Piezas de conduroy c/u. Q.3,942.40. 10 piezas de polyester c/u. Q.4,418.96. 12 Piezas de sedalina c/u. Q.4,458.16. 10 ´Piezas de gabardina c/u. Q.3,304.28. 10 Piezas de Dacron c/u. Q.4492.04. Produc tos para

De est as m erc aderías s e adeu da a:

Distribuidora de Telas S.A. Q.40,000.00 y Munditelas S.A.

Q.30,000.00.

: (Con IVA incluido) 3 Mese de alquiler pagados anticipadamente al Sr. León Peludo Manzo a Q.1,747.20 cada mes. Renta del local

el Sr. Panza Blanca: El Sr. Barco del Rio le adeuda por un préstamo que le concedió Q.4,000.00 Según recibo No. 4 y el Sr. Custodio Armas, también le adeuda por un préstamo que le concedió Q.1,500.00 recibo No. 5. (Utilice la cuenta Deudores No Comerciales y regístrelo en el activo corriente. Deudas a favor d

No. 6-767676-5 del Banco de los Trabajadores S.A. Q.45,800.00 . No-6-87766-7 del Banco Azteca por Q.38,500.00. Cuentas Bancarias:

LABORATORIO # 3 El 01 del presente mes, el Sr. Porfirio Regalón, inicia su negocio de compra y ventas de ollas, denominado Almacén “El Ollón”, le solicita le opere el libro inventario, resumen y certificación delcapital, proporcionándole la información necesaria. Utilice hojas tabulares de 3 columnas. (Con IVA incluido) OLLAS : 100 Grandes c/u. Q.54.32. 100 Medianas c/u. Q49.00. 100 Pequeñas c/u. Q.39.48. CAZOS : 120 Grandes c/u. Q.39.76; 120 Medianos c/u. Q.33.88; 120 Pequeños c/u. Q.22.96. SARTENES: 130 Grandes c/u. Q.33.88; 130 Medianos c/u. Q.27.44; 130 Pequeños c /u. Q.21.00. Produc tos para la venta:

De esta mer cad ería se adeu da sin el IVA a:

Distribuidora LaJarrilla S.A. Q.5, 000.00 y al Almacén Las

Ollas S.A. Q .4, 500.00.

No. 3-56565-5 Del Banco Agromercantil S.A. Q.60.540.00. No. 4-55555.6 del Banco Tucán S.A. Q.48,900.00. Cuentas Bancarias:

s que se vendan : (IVA Incluido) 50 cajas grandes de cartón c/u. Q5.04; 50 Cajas medianas de cartón c/u. Q.4.20; 50 cajas pequeñas de cartón c/u. Q2.52. (Utilice la cuenta Material de Empaque y regístrelo en el activo corriente). Caja s de ca rtón para empaque de los producto

Equipo : (IVA incluido) 4 Mostradores de maderas c/U. Q.728.00. 4n Estantes de metal c/u. Q.604.80. 2 Escritorios secretariales c/u. Q.649.60; 2 sillas giratorias c/u. Q.464.80; Un archivo de 4 gavetas Q.840.00; Una máquina de escribir marca Brother Q. 3,864.00; Una mesa para la máquina de escribir Q.476.00. De estos bienes se adeuda a: Muebles y Equipos Modernos S,A. 5 letras de Q.1,000.00 cada una sin IVA. Mobiliario y

Efectivo: Billetes Q.4,910.00. Monedas Q.325.00.

Desarrollo del Proceso Contable Libro Diario

El docente presenta a los estudiantes la siguienteinformación y posteriormente el ejercicio para resolverlo en clase para que ellos tengan los lineamientos a seguir para la elaboración de un libro Diario.

LIBRO DE DIARIO: Es un libro principal y obligatorio, que también recibe el nombre De primera Entrada. Se utiliza para registrar todos los movimientos y todas las transacciones comerciales de un Ente económicamente activo. En él se realizan cronológicamente partidas simples, compuestas o mixtas según la actividad que económicamente que se esté ejecutando. Se hacen uso de las cuentas según su naturaleza contable, de activo, pasivo, pérdida o ganancia, en las cuales se cargan o se abonan según los principios de partida Doble. En este libro se registran las operaciones de una empresa, siempre con base en documentos fehacientes tal como lo establece el artículo 381 del código de comercio. En este libro se describen las operaciones detalladamente, indicando no solo el nombre de las cuentas de cargo y abono, sino además, datos de carácter informativo como son: 1) Fecha de Operación o partida 2) Número de partida 3) Nombre de la empresa que interviene en la operación 4) Nombre y número de los documentos que amparan la operación. 5) Vencimiento de los documentos 6) Condiciones bajo las cuales se realizó la operación. JORNALIZACIÓN: Significa registrar, razonar, pensar o analizar que cuenta se va a cargar y qué cuenta se va a abonar.

Principios Fundamentales de la Partida Doble: 1) No hay cambio sin valor 2) No hay deudor sin acreedor 3) La cuenta deudora recibe y la cuenta acreedora da. 4) La suma del debe y haber deben ser iguales. 5) Toda partida debe llevar número, fecha y explicación. 6) Las cuentas salen con el mismo nombre que entran (14 y 15)

EJEMPLO DEL L IBRO DIARIO

Instrucciones: Con los datos que se le proporcionan elabore el libro de Diario de Comercial “ El Éxito”, propiedad del señor Juan Francisco Marroquín, el cual tiene registrado su domicilio en 2da. Avenida 3-20 zona 1 Mazatenango, Suchitepéquez durante el mes de Octubre de 2,010. Día 01)

Se invierte en el negocio Q 100,000.00

Día 05)

Se compra mercadería por valor de Q 11,500.00 en efectivo, según factura No 10 de Distribuidora La Luna.

Día 08)

Se abre una cuenta monetaria y se deposita al Banco de los Trabajadores la cantidad de Q 75,000.00, según boleta No 456825.

Día 12)

Se compra Mobiliario poro valor de Q5,000.00 A Muebles Modernos S.A. según factura No 45, el pago se realizó con cheque No 01 del Banco de los Trabajadores.

Día 15)

Se vende mercadería en efectivo por valor de Q7,000.00 según factura No 01

Día 20)

Se compra mercadería al crédito a Comercial El Mundo por valor de Q15,000.00 según factura No 852 por el total se firmaron tres pagaré.

Día 30)

Se vende mercadería al crédito por valor de Q8,000.00 al señor Francisco de León según factura No 02

pda No 1 01/10/2010 en esta fecha se inician las operaciones Contables de comercial “El Éxito” propiedad del señor Juan3-20 Francisco con domicilio en 2da. avenida zona 1Marroquín Mazatenango, Suchitepéquez. Caja a) Capital

v/ cuentas de activo y pasivo que abren al presente ejercicio contable pda No 2 05/10/2010 Compras IVA por Cobrar

100,000.00 100,000.00 100,000.00 100,000.00 10,267.86 1,232.14

a) Caja

v/ factura No 10 Distribuidora La Luna pda No 3 08/10/2010 Bancos

11,500.00 75,000.00

a) Caja

v/ Depósito No 456825 del Banco de los 75,000.00 pda No 4 Trabajadores 12/10/2010 Mobiliario y Equipo IVA por Cobrar

a) Ventas

IVA por pagar

75,000.00 75,000.00

4,464.29 535.71

a) Bancos

v/factura No 12 de Muebles Modernos S.A. se emitió cheque No 01 pda No 5 15/10/2010 Caja

11,500.00 11,500.00

5,000.00

5,000.00 5,000.00

7,000.00 6,250.00 750.00

v/factura No 01 pda No 6 20/10/2010 Compras IVA por Cobrar

7,000.00 13,392.86 1,607.14

a) Documentos por Pagar

v/factura No 852 de Comercial El Mundo pda No 7 30/10/2010 Clientes

15,000.00

pda No 8

15,000.00 15,000.00

8,000.00 7,142.86 857.14

a) Ventas IVA por Pagar

v/ factura No 2 Cliente Francisco de León

7,000.00

8,000.00

8,000.00

30/10/2010

IVA por Pagar

1,607.14

a) IVA por Cobrar

v/ regularización

1,607.14

1,607.14 1,607.14

Ejercicios de Libro Diario propuestos:

Ejercicio No 1

INSTRUCCIONES: Trabajar en papel continental de dos columnas las partidas de apertura de las empresas que a continuación sedescriben. (También realizar el Libro Diario, Mayory Libro Balance) 01.

La sociedad Pérez López y Cía. S. C., queda legalmente constituida, según escritura social el 1 de noviembre del 2011, con las siguientes aportaciones. Socio Julio Pérez aporta: Caja Q. 10, 000.00 ------- + -------- Q. Mercaderías Q. 25, 000.00 Q. Mobiliario Q. 5, 000.00 Q. Vehículos Q. Proveedores Q. 4, 000.00 Documentos por Pagar Q. 2, 500.00

Socio Pedro López aporta: 3, 500.00 12, 000.00 10, 000.00 15, 000.00 Q. 3, 500.00

02.

Venta de mercaderías al crédito Q.8, 492.50, según factura No. 0002, al Almacén Mil Colores quien nos firma un pagaré a 120 días plazo (en las operaciones esta el IVA incluido).

10.

Depositamos en el Banco City, S.A., el 60% del saldo de caja, según boleta de deposito monetario.

14.-

Compramos panel marca Toyota, modelo 2008, por valor de Q.45, 500.00, cancelamos Q.4, 000.00 en efectivo y por el resto firmamos letras de a 30, 60, 90 y 120 días a Cidea

25.

Compra de mercadería al crédito por valor de Q.3, 468.52, firmamos pagare al Almacén la fuente a 90 días Plazo.

30.

Cancelamos nomina de sueldos de administración por Q.4, 284.00, sueldos de ventas por Q.4, 548.00 con cheques No. 2-1137 al 2-1145, descontamos cuota patronal y laboral respectivamente en el mes

SOLUCION

P#1 02/03/2011 1

Caja

13,500.00

2

Mercaderías

37,000.00

3

Mobiliario y Equipo de Oficina

15,000.00

4

Vehículos

15,000.00

5

a: Proveedores

4,000.00

6

Documentos por Pagar

6,000.00

7

Capital Social

70,500.00

V/Con esta partida se inician las operaciones contables de la sociedad Pérez López & Cía. S. C., según escritura de constitución

80,500.00

80,500.00

P#2 02/03/2011 8 9 10

Documentos por Cobrar

8,492.50

a: Ventas

7,582.59

IVA por Pagar V/Factura #0002 Almacén mil colores

909.91 8,492.50

8,492.50

P#3 10/03/2011 11 1

Bancos

5,400.00

a: Caja V/Depositamos en Banco. City, S.A.,

5,400.00 5,400.00

5,400.00

P#4 14/03/2011 4

Vehículos

40,625.00

12

IVA por Cobrar

4,875.00

1 6

a: Caja Documentos por Pagar V/Factura #4555 a Cidea panel marca Toyota, modelo 2008 30,60,90,120 días

4,000.00

45,500.00

41,500.00 45,500.00

P#5 25/03/2011 13

Compras

3,096.89

12 6

IVA por Cobrar a: Documentos por Pagar

371.63

V/Factura #2525 Almacén la fuente

3,468.52

3468.52 3468.52

P#6 31/03/2011 14

Sueldos de Administracion

5,284.00

15

Sueldos de Ventas

6,548.00

16

Cuota Patronal de Administracion

669.48

17

Cuota Patronal de Ventas

829.63

18

a: Cuota Patronal por Pagar Retención cuota Laboral del IGSS

19 11

1,499.11 571.49

Bancos

11,260.51

V/Nomina # 01 Cheques #2-1137 al 2-1145 Descon Cuota laboral y patronal

13,331.11

13,331.11

Ejercicio No 2

Con los siguientes valores realizar Partidas de Diario. Valores con lo que se inicia la contabilidad del Almacén “El Progreso”, propiedad de María Santos el 01 de enero de 2011. Caja Mobiliario y equipo Mercadería Acreedores Documentos por cobrar corto plazo Clientes Proveedores Vehículos Gastos de organización e instalación Bancos Documentos por pagar largo plazo

Q117,500.00 Q5,800.00 Q100,000.00 Q5,600.00 Q9,400.00 Q4,330.00 Q5.900.00 Q50,000.00 Q20,000.00 Q50,000.00 Q25,000.00

02 de enero de 2011. Se realizó deposito en el banco de Antigua por Q10,000.00 03 de enero de 2011. Se canceló deuda total de los clientes en efectivo. 04 de enero de 2011. Se hizo abono a los proveedores por Q5,000.00 en efectivo. 05 de enero de 2011. Compra de 1 motocicleta para usos de la empresa por Q11,200.00(IVA incluido)en efectivo.

06 de enero de 2011. Compra de mercadería por Q56,000.00 pagado al contado. Venta de mercadería por Q5,600.00 en efectivo. 07 de enero de 2011. Venta de mercadería de Q5,600.00 al credito. Compra de mercadería por Q28,000.00 al crédito. 08 de enero de 2011. Venta de mercadería por Q11, 200.00 pagado 50% en efectivo y el resto al crédito. 15 de enero de 2011. Compra de mercadería por Q28, 000.00 al contado. 18 de enero de 2011. Compra de Estantería por Q1, 680.00 al contado. Compra de Computadora Q4, 480.00 al contado.

SOLUCION: Partidas de Diario del Almacén "El Progreso", propiedad de la señora María Santos, correspondiente al mes de enero de 2011. (cifras expresadas en Quetzales). Pda # 1 01 de enero de 2011. Caja Bancos Documentos por cobrar corto plazo Mercadería Clientes Gastos de Organización e instalación Vehículos Mobiliario y Equipo a) Proveedores Acreedores Documentos por pagar largo plazo Cuenta capital señor García R/ De la apertura contable de la empresa la fortuna. Pda # 2 02 de enero de 2011. Bancos a) R/ Caja De depósito en efectivo en el banco. Pda # 3 03 de enero de 2011. Caja a) Clientes R/Del pago de deuda de clientes en efectivo. Pda # 4 04 de enero de 2011. Proveedores a) Caja R/ Registro del pago a proveedores de 5,000.00 efectivo

Q

117,500.00

Q

50,000.00

Q

9,450.00

Q

100,000.00

Q

4,330.00

Q

20,000.00

Q

50,000.00

Q

5,800.00 Q

Q

357,080.00

Q

10,000.00

Q

10,000.00

Q

4,330.00

Q

4,330.00

Q

5,000.00

Q

5,000.00

5,900.00

Q

5,600.00

Q

25,000.00

Q

320,580.00

Q

357,080.00

Q Q

10,000.00 10,000.00

Q

4,330.00

Q

4,330.00

Q

5,000.00

Q

5,000.00

Pda # 5 05 de enero de 2011. Vehículos IVA por cobrar a) Caja R/ Compra de 1 motocicleta pagada en efectivo. Pda # 6 06 de enero de 2011. Compra IVA por cobrar a) Caja R/ Compra de mercadería pagada en efectivo. Pda # 7 06 de enero de 2011. Caja a) Ventas IVA por pagar R/ Venta de mercadería cobrada en efectivo. Pda # 8 07 de enero de 2011. Clientes Caja a) Ventas IVA por pagar R/ Venta de mercadería al crédito. Pda # 9 07 de enero de 2011. Compras IVA por cobrar a) Proveedores Caja R/De compra de mercadería al crédito. Pda # 10 8 de enero de 2011. Caja Clientes a) Ventas IVA por pagar R/ Venta de mercadería 50% contado y 50% crédito. Pda # 11 15 de enero de 2011. Compras IVA por cobrar a) Caja R/ Compra de mercadería pagada al contado. Pda # 12 18 de enero de 2011. Mobiliario y equipo IVA por cobrar a) Caja R/ compra de Estantería al contado. Pda # 13 18 de enero de 2011. Equipo de computación IVA por cobrar a) Caja R/Compra de Computadora al contado.

Q

10,000.00

Q

1,200.00

Q

11,200.00

Q

50,000.00

Q

6,000.00

Q

56,000.00

Q

5,600.00

Q

5,600.00

Q

5,000.00

Q

600.00

Q

5,600.00

Q

25,000.00

Q

3,000.00

Q

28,000.00

Q

6,200.00

Q

5,000.00

Q

11,200.00

Q

25,000.00

Q

3,000.00

Q

28,000.00

Q

1,500.00

Q

180.00

Q

1,680.00

Q

4,000.00

Q

480.00

Q

4,480.00

Q

11,200.00

Q

11,200.00

Q

56,000.00

Q

56,000.00

Q

5,000.00

Q

600.00

Q

5,600.00

Q

5,000.00

Q

600.00

Q

5,600.00

Q

25,000.00

Q

3,000.00

Q

28,000.00

Q

10,000.00

Q

1,200.00

Q

11,200.00

Q

28,000.00

Q

28,000.00

Q

1,680.00

Q

1,680.00

Q

4,480.00

Q

4,480.00

EJERCICIO No 3 SERIE UNICA: Indicaciones . El Gerente del Almacén La Confianza en Dios, del departamento de Suchitepéquez le pide que realice el siguiente libro Diario, el cual es propiedad de Martin Figueroa. Trabaje con fecha 01 de Enero del presente año y con la información siguiente:

RESUMEN DEL INVENTARIO

Debe

01

Caja

Q. 475,000.00

02

Bancos

Q. 630,000.00

03

IVA por Cobrar

Q. 39,214.46

04

Mercaderías

Q. 312,500.00

05

Material de Empaque

Q.

8,930.00

06

Mobiliario y Equipo

Q.

5,357.14

07

Proveedores

Q. 5,000.00

08

Capital

Q 1,466, 001. 60

Sumas Iguales

Haber

Q. 1, 471, 001. 60 Q 1, 471, 001. 60

MOVIMIENTO ECONOMICO DURANTE EL MES DE ENERO Día 05) Se compraron de Papelería y Útiles Q.3000.00 con el IVA Incluido a la Librería “ El éxito” consistente en agendas, resmas de papel, fólderes, archivadores y lapiceros. Factura No. 289, compra realizada en efectivo. Día 10) Se compra mercadería al contado por valor de Q.60,000.00 con el IVA Incluido según factura No. 078 de Sueños Dulces, S.A. Día 15) Se deposita en el banco Q.80, 000.00 en efectivo, según boleta de depósito No.145847 del banco. Día 20) Se vende mercadería por valor de Q190,000.00 en efectivo con el IVA Incluido según facturas de la No.002 a la No.015. Día 25) Se cancelan 3,000.00 de las camas matrimoniales que se debían a Merca, S.A. de manera efectiva. Recibo de caja No, 230 Día 31) Se pagan sueldos del personal por valor de Q 120,000.00 para administración, Q.190,000.00 para la sala de Ventas, también se les dio Q.10,000.00 de bonificación Incentivo a la Administración y Q.12,000.00 a la sala de ventas. Las cuotas patronales y laborales quedaron pendiente de pago. Se pago con los cheques No. 005 al 015.Planilla No.01

SOLUCION LIBRO DE DIARIO

HABILITADO: Por la Superintendencia de Administración Tributaria (S.A.T.)

Q. 0.50 C/ H.

AUTORIZADO: Por el Registro General Mercantil de República de Guatemala Q. 0.15 C/H

ALMACEN “LA CONFIANZA EN DIOS”

DIARIO

ALMACEN “LA CONFIANZA EN DIOS”

Periodo comprendido de 01-01.12 al 31.01.11 Con esta Partida se inician las Operaciones Contables del Almacén “La Confianza en Dios” propiedad del Señor Martin Figueroa, según partida de Apertura como sigue: Pda 01 -------- 01.01.12 ------Caja Bancos IVA por Cobrar

Pda 02

Pda 03

Pda 04

Pda 05

Pda 06

Pda 07

Mercaderías Material de Empaque Mobiliario y Equipo A: Proveedores Capital V/ Sumas Iguales -------- 05.01.12 ------Papelería y Útiles IVA Por Cobrar A: Caja V/ Según Factura 289 Librería El Éxito -------- 10.01.12 ------Compras IVA por Cobrar A: Caja V/ Según Factura 78 Sueños Dulces S.A. -------- 15.01.12 ------Bancos A: Caja V/ Según Boleta de deposito 145847 -------- 20.01.12 ------Caja A: Ventas IVA por Pagar V/ Según factura 002 a la 015 -------- 25.01.12 ------Proveedores A: Caja V/ Según Recibo de Caja No 230 -------- 31.01.12 ------Sueldos de Administración Sueldos Sala de Ventas Bonificación Admón. Bonificación Incentivo Incentivo de Sala Ventas Cuota Patronal de Administración Cuota Patronal Sala de Ventas A: Bancos Cuota Laboral y Patronal Por Pagar V/ Según Panilla 1, Cheque 05-15

(Cifras en Quetzales)

475,000.00 630,000.00 39,214.46 312,500.00 8,930.00 5,357.14 1,471,001.60

5,000.00 1,466,001.60 1,471,001.60

2,678.57 321.43 3,000.00

3,000.00 3,000.00

53,571.43 6,428.57 60,000.00

60,000.00 60,000.00

80,000.00 80,000.00

80,000.00 80,000.00

190,000.00 190,000.00

169,642.86 20,357.14 190,000.00

3,000.00 3,000.00

3,000.00 3,000.00

120,000.00 190,000.00 10,000.00 12,000.00 15,204.00 24,073.00 317,027.00 371,277.00

54,250.00 371,277.00

Desarrollo del Proceso Contable Libro Ma or

El docente presenta a los estudiantes la siguiente información y posteriormente el ejercicio para resolverlo en clase para que ellos tengan los lineamientos a seguir para la elaboración de un libro Diario.

LIBRO MAYOR: Es un libro principal y obligatorio, que también recibe el nombre de Centralizador, y se le llama así porque en el debe de Registrarse todas las cuentas provenientes del libro Diario, para operarlas y trasladar los saldos de cada cuenta al libro de Balances. Este libro se trabaja formalmente a Doble Folio, pero también se puede trabajar en “T” graficas o incluso en tres columnas, según el criterio contable que se utilice. (14)

EJEMPLO No 1:

LIBRO DIARIO

Instrucciones: Con los datos que se le proporcionan elabore el libro de Diario de Comercial “ El Éxito”, propiedad del señor Juan Francisco Marroquín, el cual tiene registrado su domicilio en 2da. Avenida 3-20 zona 1 Mazatenango, Suchitepéquez durante el mes de Octubre de 2,010. Día 01) Se invierte en el negocio Q 100,000.00 Día 05)

Se compra mercadería por valor de Q 11,500.00 en efectivo, según factura No 10 de Distribuidora La Luna.

Día 08)

Se abre una cuenta monetaria y se deposita al Banco de los Trabajadores la cantidad de Q 75,000.00, según boleta No 456825.

Día 12)

Se compra Mobiliario poro valor de Q5,000.00 A Muebles Modernos S.A. según factura No 45, el pago se realizó con cheque No 01 del Banco de los Trabajadores.

Día 15)

Se vende mercadería en efectivo por valor de Q7,000.00 según factura No 01

Día 20)

Se compra mercadería al crédito a Comercial El Mundo por valor de Q15,000.00 según factura No 852 por el total se firmaron tres pagaré.

Día 30)

Se vende mercadería al crédito por valor de Q8,000.00 al señor Francisco de León según factura No 02

pda No 1 01/10/2010 en esta fecha se inician las operaciones Contables de comercial “El Éxito” propiedad del señor Juan Francisco Marroquín con domicilio en 2da. avenida 3-20 zona 1 Mazatenango, Suchitepéquez. Caja b) Capital

v/ cuentas de activo y pasivo que abren al presente ejercicio contable pda No 2 05/10/2010 Compras IVA por Cobrar

100,000.00 100,000.00 100,000.00 100,000.00 10,267.86 1,232.14 11,500.00

b) Caja

v/ factura No 10 Distribuidora La Luna pda No 3 08/10/2010 Bancos

11,500.00 75,000.00

b) Caja

v/ Depósito No 456825 del Banco de los 75,000.00 Trabajadores pda No 4 12/10/2010 Mobiliario y Equipo 4,464.29 IVA por Cobrar 535.71 b) Bancos

v/factura No 12 de Muebles Modernos S.A. se emitió cheque No 01 pda No 5 15/10/2010 Caja

5,000.00

7,000.00

v/ factura No 2 Cliente Francisco de León

750.00 7,000.00

13,392.86 1,607.14 15,000.00

15,000.00 15,000.00

8,000.00 7,142.86 857.14

b) Ventas IVA por Pagar

pda No 8

5,000.00 5,000.00

6,250.00

b) Documentos por Pagar

v/factura No 852 de Comercial El Mundo pda No 7 30/10/2010 Clientes

75,000.00 75,000.00

7,000.00

b) Ventas

IVA por pagar v/factura No 01 pda No 6 20/10/2010 Compras IVA por Cobrar

11,500.00

8,000.00

8,000.00

30/10/2010

IVA por Pagar

1,607.14

b) IVA por Cobrar

v/ regularización

1,607.14

1,607.14 1,607.14

LI BRO MAYO R CAJA

01/10/2010 01/10/2010

a) Capital a) Varios

100,000.00 107,000.00

05/10/2010 08/10/2010

por) Varios por) Bancos

75,000.00

11,500.00 86,500.00

107,000.00

30/10/2010

por) Saldo

20,500.00

107,000.00

01/10/2010

7,000.00

CAPITAL

30/10/2010

a) Saldo

100,000.00 100,000.00

05/10/2010 20/10/2010

a) Caja a) Doctos. Por pagar

10,267.86 23,660.72 23,660.72

por) Caja

100,000.00 100,000.00

COMPRAS

13,392.86

30/10/2010

por) Saldo

23,660.72 23,660.72

IVA POR CO BRA R

05/10/2010 12/10/2010 20/04/2010

a) caja a) Bancos a) Doctos. Por pagar

535.71 1,607.14

1,232.14 1,767.85 3,374.99 3,374.99

30/10/2010

por) Iva por pagar por) Saldo

1,767.85

1,607.14 3,374.99 3,374.99

BANCOS

08/10/2010

a) Caja

75,000.00

08/10/2010 30/10/2010

por) Varios por) Saldo

30/10/2010

por) Saldo

70,000.00

5,000.00 75,000.00 75,000.00

MOBILIARIO Y EQUIPO

12/10/2010

a) Bancos

4,464.29 4,464.29

4,464.29 4,464.29

VENTAS

08/10/2010

a) Saldo

13,392.86

15/10/2010 30/10/2010

por) Caja por) Clientes

7,142.86

6,250.00 13,392.86 13392.86

por) Caja por) Clientes

857.14

750.00 1.607.14 1.607.14

13,392.86 IVA POR PAGAR

08/10/2010

a) Iva por cobrar

1,607.14

15/10/2010 30/10/2010

1,607.14 DOCUMENTOS POR PAGAR

12/10/2010

a) Saldo

15,000.00 15,000.00

20/10/2010

12/10/2010

a) Varios

8,000.00 8,000.00

30/10/2010

por) Varios

15,000.00 15,000.00

por) Saldo

8,000.00 8,000.00

CLIENTES

EJERCICIOS PROPUESTOS DE LIBRO MAYOR:

EJERCICIO No 1 Información proveniente del ejercicio No 2 del Libro Diario Libro mayor del Almacén "El Progreso", propiedad de la señora , María Santos, correspondientes al mes de enero de 2011. ( Cifras expresadas en Quetzales).

Caja

Documentos por cobrar corto plazo

pda 1

Q

117,500.00

Q

pda 3

Q

4,330.00

Q

5,000.00 pda 4

pda 7

Q

5,600.00

Q

11,200.00 pda 5

pda 8

Q

600.00

Q

56,000.00 pda 6

pda10 Q

6,200.00

Q

3,000.00 pda 9

Q

28,000.00 pda11

Q

1,680.00 pda 12

Q

4,480.00 pda 13

Q

134,230.00

Q

14,870.00

pda 1

Q

50,000.00

pda 2

Q

10,000.00

Q Q

Q

10,000.00 pda 2 pda 1

119,360.00

Bancos

Q

9,450.00

Q

9,450.00

Q

9,450.00

Mercadería pda 1

Q

100,000.00

60,000.00

Q

100,000.00

60,000.00

Q

100,000.00

Clientes

Gastos de organización e instalación

pda 1

Q

4,330.00

pda 8

Q

5,000.00

pda10 Q

5,000.00

Q

14,330.00

Q

10,000.00

Q

4,330.00 pda 3 pda 1

Q

20,000.00

Q

4,330.00

Q

20,000.00

Q

20,000.00

Vehículos

Mobiliario y equipo

pda 1

Q

50,000.00

pda 1

Q

5,800.00

pda5

Q

10,000.00

pd12

Q

1,500.00

Q

60,000.00

Q

7,300.00

Q

60,000.00

Q

7,300.00

Proveedores pda 4

Q

Q

5,000.00

5,000.00

Acreedores Q

5,900.00 pda 1

Q

25,000.00 pda 9

Q

5,600.00 pda1

Q

30,900.00

Q

5,600.00

Q

25,900.00

Q

5,600.00

Q

320,580.00

Q

320,580.00

Q

320,580.00

Documentos por pagar largo plazo Q

25,000.00 pda 1

Q

25,000.00

Q

25,000.00

Cuenta capital

Q

IVA

Compras

pda 5

Q

1,200.00

Q

600.00 pda 7 pda 6

Q

50,000.00

pda 6

Q

6,000.00

Q

600.00 pda 8 pda 9

Q

25,000.00

pda 9

Q

3,000.00

Q

1,200.00 pd10 pda11 Q

25,000.00

pda11 Q

3,000.00

pda12 Q

180.00

pda13 Q

480.00

Q

13,860.00

Q

2,400.00

11,460.00

Q

100,000.00

Q

100,000.00

Ventas

Equipo de computación Q

5,000.00 pda 7 pda13 Q

Q

5,000.00 pda 8

4,000.00

Q

10,000.00 pda 10

Q

20,000.00

Q

4,000.00 Q

Q

20,000.00

Q

4,000.00 Q

EJERCICIO No 2 Información proveniente del ejercicio No 3 del Libro Diario

LIBRO MAYOR (En T Graficas)

BANCOS (2)

CAJA (1) DEBE 475,000.00 190,000.00

HABER 3,000.00 60,000.00 80,000.00 3,000.00

665,000.00

146,000.00

519,000.00 (S.) 665,000.00

ALMACEN “LA CONFIANZA EN DIOS” DEBE 630,000.00 80,000.00 710,000.00

HABER 317,027.00

317,027.00

392,973.00 (S.) 710,000.00

710,000.00

665,000.00

IVA POR COBRAR (3)

MERCADERIAS (4)

DEBE 39,214.46 321.43 6,428.57

HABER

DEBE 312,500.00

HABER

312,500.00

000,000.00

45,964.46

00,000.00

45, 964.46

45, 964.46 (S.) 45, 964.46

312,500.00 (S.) 312,500.00

312,500.00

MATERIAL DE EMPAQUE (5)

MOBILIARIO Y EQUIPO (6)

DEBE 8,930.00

HABER

DEBE 5,357.14

8,930.00

0,000.00

5,357.14

0,000.00

8,930.00

8,930.00 (S.) 8,930.00

5,357.14

5,357.14 (S.) 5,357.14

HABER

PROVEEDORES (7)

CAPITAL (8)

DEBE 3,000.00

HABER 5,000.00

DEBE

HABER 1,466,001.60

5,000.00

0,000,000.00

1,466,001.60

5,000.00

1, 466,001.60 (S) 1, 466,001.60

1, 466,001.60

3,000.00

2,000.00 (S.) 5,000.00

PAPELERIA Y UTILES (09)

COMPRAS (10)

2,678.57

0,000.00

53,571.43

00,000.00

2,678.57

2,678.57 (S.) 2,678.57

53,571.43

53,571.43 (S.) 53,571.43

DEBE 2,678.57

HABER

DEBE 53,571.43

HABER

IVA POR PAGAR (12)

VENTAS (11) DEBE

HABER 169,642.86

DEBE

HABER 20,357.14

000,000.00

169,642.86

00,000.00

20,357.14

169,642.86 (S) 169,642.86

169,642.86

20,357.14 (S.) 20,357.14

20,357.14

SUELDOS DE ADMON (13)

SUELDOS SALA DE VENTAS (14)

DEBE 120,000.00

HABER

DEBE 190,000.00

120,000.00

000,000.00

190,000.00

120,000.00

120,000.00 (S.) 120,000.00

HABER

000,000.00

190,000.00 (S.) 190,000.00

190,000.00

BONIF. INCENTIVO DE ADMON (15)

BONIF. INCENTIVO SALA VENTAS (16)

DEBE 10,000.00

HABER

DEBE 12,000.00

10,000.00

00,000.00

12,000.00

10,000.00

00,000.00

12,000.00 (S.)

10,000.00 (S.) 10,000.00

HABER

12,000.00

12,000.00

CUAOTA PATRONAL DE ADMON(17)

CUOTA PATRONAL SALA VENTAS (18)

DEBE 15,204.00

HABER

DEBE 24,073.00

HABER

00,000.00

24,073.00

00,000.00

15,204.00

24,073.00 (S.)

15,204.00 (S.) 15,204.00

15,204.00

CUOTA LAB. Y PAT. POR PAGAR (19) DEBE

HABER 54,250.00

00,000.00

54,250.00

54,250.00 (S.) 54,250.00

54,250.00

24,073.00

24,073.00

Desarrollo del Proceso Contable Libro Balance

LIBRO DE BALANCES: Es un libro principal y obligatorio, que también recibe el nombre De Estados Financieros. En el se registra el Balance General de Apertura, los Balances de Saldos ordinarios y extraordinarios, y los Estados Financieros ( estado de Resultado ydeBalance General). Se leenllama de Saldos porqueSeallítrabaja se registran todos en los un saldos las cuentas utilizadas nuestra contabilidad. formalmente libro de cuatro columnas y se puede manejar debe, haber, Deudor y Acreedor, pero también dependerá del criterio contable que se maneje siempre y cuando estén dentro de los principios y normas de contabilidad generalmente aceptados.

EJEMPLO DEL LIBRO BALANCE LIBRO DIARIO Instrucciones: Con los datos que se le proporcionan elabore el libro de Diario de Comercial “ El Éxito”, propiedad del señor Juan Francisco Marroquín, el cual tiene registrado su domicilio en 2da. Avenida 3-20 zona 1 Mazatenango, Suchitepéquez durante el mes de Octubre de 2,010. Día 01)

Se invierte en el negocio Q 100,000.00

Día 05)

Se compra mercadería por valor de Q 11,500.00 en efectivo, según factura No 10 de Distribuidora La Luna.

Día 08)

Se abre una cuenta monetaria y se deposita al Banco de los Trabajadores la cantidad de Q 75,000.00, según boleta No 456825.

Día 12)

Se compra Mobiliario para valor de Q5,000.00 A Muebles Modernos S.A. según factura No 45, el pago se realizó con cheque No 01 del Banco de los Trabajadores.

Día 15) Día 20)

Se vende mercadería en efectivo por valor de Q7,000.00 según factura No 01 Se compra mercadería al crédito a Comercial El Mundo por valor de Q15,000.00 según factura No 852 por el total se firmaron tres pagaré.

Día 30)

Se vende mercadería al crédito por valor de Q8,000.00 al señor Francisco de León según factura No 02

pda No 1 01/10/2010 en esta fecha se inician las operaciones Contables de comercial “El Éxito” propiedad del señor Juan Francisco Marroquín con domicilio en 2da. avenida 3-20 zona 1 Mazatenango, Suchitepéquez. Caja c) Capital

v/ cuentas de activo y pasivo que abren al presente ejercicio contable pda No 2 05/10/2010 Compras IVA por Cobrar

100,000.00 100,000.00 100,000.00 100,000.00 10,267.86 1,232.14

c) Caja

v/ factura No 10 Distribuidora La Luna pda No 3 08/10/2010 Bancos

11,500.00 75,000.00

c) Caja

v/ Depósito No 456825 del Banco de los 75,000.00 Trabajadores pda No 4 12/10/2010 Mobiliario y Equipo 4,464.29 IVA por Cobrar 535.71 c) Bancos

v/factura No 12 de Muebles Modernos S.A. se emitió cheque No 01 pda No 5 15/10/2010 Caja

5,000.00 7,000.00

c) Ventas

IVA por pagar v/factura No 01 pda No 6 20/10/2010 Compras IVA por Cobrar

7,000.00

15,000.00

v/ factura No 2 Cliente Francisco de León

5,000.00 5,000.00

6,250.00 750.00 7,000.00

15,000.00 15,000.00

8,000.00 7,142.86 857.14

c) Ventas IVA por Pagar

pda No 8

75,000.00 75,000.00

13,392.86 1,607.14

c) Documentos por Pagar

v/factura No 852 de Comercial El Mundo pda No 7 30/10/2010 Clientes

11,500.00 11,500.00

8,000.00

8,000.00

30/10/2010

IVA por Pagar

1,607.14

c) IVA por Cobrar

v/ regularización

1,607.14

1,607.14 1,607.14

L I BRO M AYO R CAJA

01/10/2010 01/10/2010

a) Capital a) Varios

100,000.00

05/10/2010

107,000.00

08/10/2010

7,000.00 107,000.00

CAPITAL

30/10/2010

a) Saldo

100,000.00 100,000.00

30/10/2010

01/10/2010

por) Varios por) Bancos por) Saldo

11,500.00 86,500.00 75,000.00 20,500.00

por) Caja

107,000.00

100,000.00 100,000.00

COMPRAS

05/10/2010

a) Caja

30/10/2010

por) Saldo

10,267.86 20/10/2010

a) Doctos. 13,392.86 Por pagar

23,660.72

23,660.72 23,660.72

23,660.72

IVA POR COBRA R

05/10/2010

a) caja

1,232.14

12/10/2010

a) Bancos

20/04/2010

a) Doctos. 1,607.14 Por pagar

535.71

30/10/2010

por)

Iva

por pagar por) Saldo

1,767.85

1,607.14 1,767.85

3,374.99

3,374.99 3,374.99

3,374.99

BANCOS

08/10/2010

a) Caja

75,000.00

08/10/2010 30/10/2010

por) Varios por) Saldo

70,000.00

5,000.00 75,000.00 75,000.00

MOBILIARIO Y EQUIPO

12/10/2010

a) Bancos

4,464.29 4,464.29

30/10/2010

por) Saldo

4,464.29 4,464.29

VENTAS

08/10/2010

a) Saldo

13,392.86 13,392.86

15/10/2010 30/10/2010

por) Caja por) Clientes

7,142.86

6,250.00 13,392.86 13392.86

IVA POR PAGA R

08/10/2010

a) Iva por cobrar

1,607.14

15/10/2010

por) Caja

30/10/2010

por) Clientes

750.00 857.14

1,607.14 DOCUMENTOS POR

12/10/2010

1.607.14 1.607.14

PAGA R

a) Saldo

15,000.00 15,000.00

20/10/2010

por) Varios

15,000.00 15,000.00

a) Varios

8,000.00 8,000.00

30/10/2010

por) Saldo

8,000.00 8,000.00

CLIENTES

12/10/2010

LI BRO DE BALANCE Caja 01

02 03 04 05 06 07 08 09

Capital Compras IVA por Cobrar Bancos Mobiliario y Equipo Ventas Documentos por Pagar Clientes

20,500.00 100,000.00 23,660.72 1,767.85 70,000.00 4,464.29 13,392.86 15,000.00 8,000.00 128,392.86

128,392.86

EJERCICIOS PROPUESTOS DEL BALANCE DE SALDOS:

EJERCICIO No 1 Información Proveniente del ejercicio No 2 del Libro Mayor

Balance de Saldos. Almacén "El Progreso". Al 31 de enero de 2011. (Cifras expresadas en Quetzales). Debe

Haber

Caja Bancos Documentos por cobrar corto plazo Mercadería Clientes IVA por cobrar Gastos de organización e instalación

Q

14,870.00

Q

60,000.00

Vehículos Mobiliario y equipo Equipo de computación Compras Proveedores Acreedores Documentos por pagar a largo plazo Ventas Cuenta Capital señora Méndez Sumas iguales

Q

9,450.00

Q

100,000.00

Q

10,000.00

Q

11,460.00

Q

20,000.00

Q Q

60,000.00 7,300.00

Q

4,000.00

Q

100,000.00

Q

397,080.00

EJERCICIO No 2

Información proveniente del ejercicio No 3 del Libro Mayor

Q

25,900.00

Q

5,600.00

Q

25,000.00

Q

20,000.00

Q

320,580.00

Q

397,080.00

LIBRO DE BALANCES

HABILITADO:

ALMACEN “LA CONFIANZA EN DIOS”

Por la Superintendencia de

Administración Tributaria (S.A.T.) Q. 0.50 C/ H.

Por el Registro General Mercantil de República de Guatemala Q. 0.15 C/H AUTORIZADO:

BALANCE DE SITUACION GENERAL DE APERTURA

ALMACEN “LA CONFIANZA EN

DIOS”

No

Al 01 de Enero del 2,012 Cuentas

(Cifras en Quetzales)

ACTIVO CORRIENTE

Caja Bancos IVA por Cobrar

475,000.00 630,000.00 39,214.46

Mercaderías Material de Empaque

312,500.00 8,930.00

1,465,644.46

NO CORRIENTE

Mobiliario y Equipo SUMA DEL ACTIVO

5,357.14 1,471,001.60

PASIVO CORRIENTE

Proveedores SUMA DEL PASIVO

5,000.00 5,000.00

PATRIMONIO NETO

1,466,001.60

CAPITAL

Suma

del

Pasivo y

Guatemala,

Martin Figueroa Propietario

Patrimonio

01

de

Neto

Enero de

Rodolfo

1,471,001.60

2,012

Tùnchez P.C.

García

BALANCE DE COMPROBACION

ALMACEN “LA CONFIANZA EN DIOS”

Comprendido del 01.01.12 al 31.01.12 No 01 02 03 04 05 06 07 08 09 10 11 12 13 14 15 16 17 18 19 

(Cifras en Quetzales) SUMAS SALDOS Cuentas DEBE HABER DEUDOR ACREEDOR Caja 665,000.00 146,000.00 519,000.00 Bancos 710,000.00 317,027.00 392,973.00 IVA por Cobrar 45,964.46 45,964.46 Mercaderías 312,500.00 312,500.00 Material de Empaque 8,930.00 8,930.00 Mobiliario y Equipo 5,357.14 5,357.14 Proveedores 3,000.00 5,000.00 2,000.00 Capital 1,466,001.60 1,466,001.60 Papelería y Útiles 2,678.57 2,678.57 Compras 53,571.43 53,571.43 Ventas 169,642.86 169,642.86 IVA por Pagar 20,357.14 20,357.14 Sueldos de Admón. 120,000.00 120,000.00 Sueldos Sala de Ventas 190,000.00 190,000.00 Bonif. Incen. De Admón. 10,000.00 10,000.00 Bonif. Incen. Sala Ventas 12,000.00 12,000.00 Cuota Patronal Admón. 15,204.00 15,204.00 Cuota Patron. Sala Venta 24,073.00 24,073.00 Cuota Lab. Y Pat X pagar 54,250.00 54,250.00 2,178,278.60 2,178,278.60 1,712,251,60 1,712,251.60 Sumas Iguales

Guatemala,

Martin Figueroa Propietario

31

de

Enero de

Rodolfo

2,012

Tùnchez P.C.

García

Cierre del Proceso Contable La Contabilidad como fuente de información ara la toma de decisiones ersonales em resariales La o el docente proporcionará un folleto a los alumnos, el cual copiarán a su cuaderno y en una clase posterior darán un diagnóstico sobre la toma de decisiones teniendo como fuente de información La Contabilidad de una empresa.

FOLLETO: Estado de Resultados: Es un Estado Financiero que se le conoce también como el Estado de Pérdidas y Ganancias, el cual se trabaja en el Libro Obligatorio de Balances. En el se establecen si el ente económicamente obtuvo ganancia o pérdida durante su ejercicio fiscal. El Estado de Resultados forma parte de los Estados Financieros de un negocio, el cual presenta en forma ordenada los ingresos, costos y gastos de la empresa referidos a un período determinado, que puede ser un mes o un año. El Estado de Resultados sirve para dar a conocer el importe de ganancias o pérdidas. Principales requisitos que deben incluir Los Estados Financieros: 1) El nombre, u otro tipo de identificación de la empresa. 2) Si es empresa individual o pertenece a una sociedad. 3) El período que cubre o la fecha de cierre, según el Estado Financiero de que se trate. 4) La moneda en que se presenta la información. 5) El nivel de precisión utilizado en la presentación de las cifras. Balance General: Es un cuadro contable que contiene una lista de cuentas utilizadas en el libro mayor y nos muestra la situación de los activos, pasivos y capital de un negocio. El período contable de toda empresa o negocio es de un año. En Guatemala tradicionalmente el período contable se inicia el 1ro de Julio de un año y concluye el 30 de Junio del siguiente año; del 1ro de enero al 31 de diciembre de un año, o cualquier otro período autorizado por la SAT. El cierredede operaciones de un laejercicio contable las cuentas del Estado Resultados, mediante jornalización, queimplica consistesaldar en cargar las cuentas de Pérdidas y Ganancias y abonar todas las cuentas de Costos y Gastos de la empresa, y cargar todas las cuentas de ingreso y productos, abonando la cuenta de Pérdidas y Ganancias. El objeto de saldar o efectuar las partidas de cierre se debe a que las cuentas de resultados de Ingresos y Gastos no pueden acumular valor para el período siguiente, conforme lo establece el principio de Periodo Contable.

El cierre de Balance General se hará mediante la jornalización de las partidas de cierre, así: Las cuentas del Balanace General que son Acreedoras se cargan y las cuentas Deudoras se abonan; al igual que las cuentas del Estado de Resultados al final de un período contable se deben saldar o cerrar. MEDICION DE LOS ELEMENTOS DE LOS ESTADOS FINANCIEROS: Medición es el proceso de determinación de los importes monetarios por los que se reconocen y Llevan contablemente los elementos de los estados financieros, para su inclusión en el balance y el estado de resultados. Para realizarla es necesario la selección de una base o método particular de medición. En los Estados Financieros se emplean diferentes bases de medición con diferentes grados y en distintas combinaciones entre ellas. Tales bases o métodos son los siguientes: a) Costo Histórico: Los activos se registran por el importe de efectivo y otras partidas pagadas, o por el valor razonable de la contrapartida entregada a cambio en el momento de la adquisición. b) Costo Corriente: Los activos se llevan contablemente por el importe de efectivo y otras partidas equivalentes al efectivo, que debería pagarse si se adquiriese en la actualidad el mismo activo u otro equivalente. c) Valor Realizable: (o de liquidación) Los activos se llevan contablemente por el importe del efectivo y otras partidas equivalentes al efectivo que podrían ser obtenidos, en el momento presente, por la venta no forzada de los mismos. d) Valor presente: Los activos se llevan contablemente al valor presente, descontando las entradas netas de efectivo que se espera que genere la partida en curso normal de la operación.

Luego del desarrollo de las actividades sugeridas y aquellas diseñadas por usted, se sugiere que evalúe el proceso de aprendizaje, para verificar el nivel de alcance de la competencia No. 4

Heteroevaluación. Es el procedimiento que realiza el docente para evaluar el nivel de aprendizaje de las y los estudiantes. Se recomienda aplicar los siguientes ejercicios. Por ser casos prácticos se recomienda al docente que realice este tipo de evaluación. A continuación se le presentan algunos ejercicios, los cuales podría utilizar como evaluaciones para conocer el nivel de aprendizaje de la o el estudiante.

EVALUACIÓN ACTIVIDAD No 5 Apellidos y Nombres:_________________________________________________ Fecha:___________________________ Punteo:___________________________ Con los datos que se le presentan elabore el Inventario No 01 de Comercial “ La Esperanza” propiedad del señor José Manolo Abraham Valiente Arango ubicado en 3ra calle 2-21 zona 1, Río Bravo, Suchitepéquez.

Para el efecto se proporciona la siguiente información: Posee en efectivo Q 5,000.00 Cuenta Bancaria No 346456-9 Banco Agromercantil Q 15,000.00 Muebles para uso de la empresa: 1 estantería de metal marca fuerte Q 800.00 1 Escritorio de metal marca Buen Metal Q 1,500.00 Artículos para la venta: 10 pantalones Marca Lee talla 34 cada uno Q 350.00 15 pantalones Marca Pier Cardin talla 36 cada uno Q 290.00 10 pantalones Marca Lee Talla 32 cada uno Q 340.00 05 playeras marca Lee talla M cada una Q 150.00 La mercadería posee el IVA incluido. De la mercadería se deben los pantalones Pier Cardin a Distribuidora Promesa. Se debe al Banco El Águila Q 3,000.00 por un préstamo que nos concedió a tres años plazo.

EVALUACIÓN ACTIVIDAD No 6 Apellidos y Nombres:_________________________________________________ Fecha:___________________________ Punteo:___________________________

El señor Milamores Tedoy inició un negocio de compra y venta ropa para dama llamado Almacén “Vista Cómoda”, le solicita a usted elabore el libro Diario para lo cual le proporciona la información del 1 de Junio de 2011 de Inventario No. 1 Caja Q. 3,500.00, Bancos Q. 15,000.00, Mercaderías Q. 25,000.00, Mobiliario y equipo Q. 4,200.00, Vehículos Q. 30,000.00, Inmuebles Q.216,000.00, Proveedores Q. 7,500.00, Hipotecas Q. 180,000.00, Documentos por pagar Q. 2,100.00, Acreedores Q. 1,600.00 Capital Q. 102,500.00 Día

04 Se vendió mercadería por un valor de Q. 1,600.00 al contado según factura No. 01

Día 05 Se vendió mercadería por un valor de Q. 1,870.00 al sr. Joel Montenegro según factura No. 13 el IVA se cobró el efectivo. Día 09 Se compra mercadería por un valor de Q3,000.00 para lo cual firmamos una letra de cambio a 90 días a favor de Distribuidora La Paz, según factura. No. 6879. El IVA lo pagamos con el cheque No. 45 Día 12 Se compró mercadería al contado porvalor de Q. 950.00 fact. No. 16 Día 23 Se pagó Q. 250.00 por publicidad a Nuestro Periódico con cheque No. 03 según factura No. 453

Día 30 Se pagaron sueldos por Q. 6,200.00 se retuvo la cuota del I.G.S.S. se emitió cheque No. 02

SOLUCION DE LA EVALUACION

LIBRO DIARIO

DEBE

HABER

01-06-2011 1 Caja Bancos Mercaderías Mobiliario y Equipo Vehículos Inmuebles

3,500.00 15,000.00 25,000.00 4,200.00 30,000.00 216,000.00

a) Proveedores

Hipotecas Documentos por Pagar Acreedores Capital v/ con esta partida inicia operaciones contables 2 04-06-2011 Caja

293,700.00 1,600.00

1,428.57 171.43

a) Ventas

Iva por pagar 3

v/venta de mercadería según factura No. 01 05-06-2011 Clientes Caja

1,600.00

1,870.00

Compras Iva por cobrar

3,000.00

950.00

950.00 950.00

223.21 26.79

a) Bancos

v/pago de publicidad a Periódico factura. No. 453

2,678.57 321.43 3,000.00

848.21 101.79

a) Caja

v/compra de mercadería según factura No. 16 6 23-06-2011 Publicidad Pagada Iva por cobrar

1,669.64 200.36 1,870 .00

2,678.57 321.43

a) Documentos por pagar

Bancos v/compra de mercadería según factura No. 6879 5 12-06-2011

1,600.00

1,669.64 200.36

a) Ventas

Iva por pagar v/venta de mercadería según factura No. 13 4 09-06-2011 Compras Iva por cobrar

7,500.00 180,000.00 2,100.00 1,600.00 102,500.00 293,700.00

250.00

250.00 250.00

7

30-06-2011 Sueldos Pagados

6,200.00 299.46 5,900.54 6,200.00

a) Cuota Laboral I.G.S.S.

Bancos v/pago de sueldos cheque No. 02

6,200.00

EVALUACION ACTIVIDAD No 7 Apellidos y Nombres:_________________________________________________ Fecha:___________________________ Punteo:___________________________ El señor Milamores Tedoy inició un negocio de compra y venta ropa para dama llamado Almacén “Vista Cómoda”, le solicita a usted elabore el libro Mayor para lo cual le proporciona la información del 1 de Junio de 2011 de Inventario No. 1 y la información del Libro Diario del mes de Junio (ver página anterior )

SOLUCIÓN DE LA EVALUACIÓN LIBRO MAYOR DEBE 1

2

3 4 5 6

Caja a varios part. 1/1 a varios part. 2/1 a varios part. 3/1 Por varios part. 5/1 Bancos a varios par. 1/1 Por varios part. 4/1 Por varios part. 6/1 Por varios part. 7/1 Mercaderías a varios part. 1/1 Mobiliario y Equipo a varios part. 1/1 Vehículos a varios part. 1/1 Inmuebles a varios part. 1/1

HABER

3,500.00 1,600.00 200.36

SALDO

950.00

3,500.00 5,100.00 5,300.36 4,350.36

321.43 250.00 5,900.54

15,000.00 14,678.57 14,428.57 8,528.03

15,000.00

25,000.00

25,000.00

4,200.00

4,200.00

30,000.00

30,000.00

216,000.00

216,000.00

7

Proveedores Por varios part. 1/1 Hipotecas Por varios part. 1/1 Documentos por pagar Por varios part. 1/1 Por varios part. 4/1 Acreedores Por varios part. 1/1

7,500.00

7,500.00

180,000.00

180,000.00

2,100.00 2,678.57

2,100.00 4,778.57

1,600.00

1,600.00

11 Capital Por varios part. 1/1 12 Ventas Por caja part. 2/1

102,500.00

102,500.00

1,428.57

1428.57

Por varios part. 3/1 Iva por pagar Por caja part. 2/1 Por varios part. 3/1 14 clientes a varios part. 3/1 15 Compras a varios part. 4/1 a caja part. 5/1 16 Iva por cobrar

1,669.64

3,098.21

171.43 200.36

171.43 371.79

8 9 10

13

a varios part. 4/1 a caja part. 5/1 a bancos part. 6/1 17 Publicidad pagada a bancos part. 6/1 18 Sueldos pagados a varios part.7/1 19 Cuota Laboral I.G.S.S. Por sueldos pagados part 7/1

1,669.64

1,669.64

2,678.57 848.21

2,678.57 3,526.78

321.43 101.79 26.79

321.43 423.22 450.01

223.21

223.21

6,200.00

6,200.00 299.46

299.46

EVALUACION ACTIVIDAD No 8 Apellidos y Nombres:_________________________________________________ Fecha:___________________________ Punteo:___________________________ El señor Milamores Tedoy inició un negocio de compra y venta ropa para dama llamado Almacén “Vista Cómoda”, le solicita a usted elabore el libro Balance para lo cual le proporciona la información del 1 de Junio de 2011 de Inventario No. 1 y la información del Libro Diario y Libro Mayor del mes de Junio (ver página anterior )

SOLUCION DE LA EVALUACIÓN LIBRO BALANCE 1 2 3 4 5

Caja Bancos Mercaderías Mobiliario y Equipo Vehículos

67 8 9 10 11 12 13 14 15 16 17 18 19

Inmuebles Proveedores Hipotecas Documentos por pagar Acreedores Capital Ventas Iva por pagar clientes Compras Iva por cobrar Publicidad pagada Sueldos pagados Cuota Laboral I.G.S.S. SUMAS IGUALES

Bibliografía:

DEBE 5,300.36 15,000.00 25,000.00 4,200.00 30,000.00

HABER 950.00 6,471.97

DEUDOR 4,350.36 8,528.03 25,000.00 4,200.00 30,000.00

ACREEDOR

216,000.00

7,500.00 180,000.00 4,778.57 1,600.00 102,500.00 3,098.21 371.79

216,000.00

7,500.00 180,000.00 4,778.57 1,600.00 102,500.00 3,098.21 371.79

1,669.64 3,526.78 450.01 223.21 6,200.00 307,570.00

1,669.64 3,526.78 450.01 223.21 6,200.00 299.46 307,570.00

300,148.03

299.46 300,148.03

9) Contabilidad General de Editora Educativa, páginas 96 y 97 10) Contabilidad General, Tercero Básico, EDITEXSA, página 3 11) Contabilidad de Sociedades, Alfredo Enrique Ruíz Orellana 12) Contabilidad de Sociedades de Editora Educativa 13) Contabilidad General, Stella Castillo Milla, Edición Alenro, página 91 14) Contabilidad de Sociedades, Alfredo Enrique Ruíz Orellana

MINISTERIO DE EDUCACION Ciclo Escolar: ___2011________ Establecimiento educativo: INSTITUTO NACIONAL DE EDUCACION BASICA Grado: TERCERO BÁSICO Nombre del o la docente: _____________________________________________ AREA/DESARROLLO Y PRODUCTIVIDAD / SUBAREA: CONTABILIDAD III.

FORMATO PARA LA ORGANIZACIÓN DEL TRABAJO DOCENTE BLOQUE 1 COMPENTENCIA CONTENIDO DECLARATIVO 1. Formula proyectos variables que Regímenes de Tributación. propician el mejoramiento escolar o comunitario.  Definición. 

Impuestos



Formularios

Sistema Tributario Nacional  

Concepto de Impuestos

 

Clasificación de Impuestos.

 

Tarifas de Impuestos

 

Tipos de Contribuyentes

 

Educación Fiscal

BLOQUE 2 CONTENIDO DECLARATIVO

COMPENTENCIA 2. Aplica técnicas de mejora continua en el desarrollo de procesos Presupuestos de una empresa productivos en su entorno.  Metodología presupuestal 

Concepto 

Tipos de presupuestos 

Origen



Integración 

Preparación, elaboración y ejecución. 

Análisis de las variaciones presupuestarias. 

Técnicas de registro

BLOQUE 3 COMPENTENCIA CONTENIDO DECLARATIVO Ejecuta propuestas de emprendimiento productivo escolar o 1.-Contabilidad y técnicas de registro en libros de Contabilidad comunitario que enfatiza la Concepto integración de liderazgo y procesos administrativos y comerciales. Principio de Contabilidad Tipos de Contabilidad Base Legal Libros Obligatorios de Contabilidad Libros Auxiliares de Contabilidad Técnicas de Registro Terminología Contable

2.- Apertura de la Contabilidad Libro Inventario Concepto Integración Balance General de Apertura

Ejecución

3.- Libro Diario y Libro Caja Concepto Jornalizar Partidas Simples Partidas Mixtas

Cálculo del IVA

Partidas compuestas 4.- Libro Mayor y Libro Balance Libros Auxiliares del Mayor Libro Compras Libro Ventas 5.- La Contabilidad como fuente de información para el planteamiento y toma de decisiones personales y empresariales. Proceso Contable Etapas del Proceso Contable Cierre Contable

View more...

Comments

Copyright ©2017 KUPDF Inc.
SUPPORT KUPDF